ChaseDream
标题: 【每日逻辑练习第二季】【2-1】 [打印本页]
作者: corneliaflower 时间: 2011-12-9 22:57
标题: 【每日逻辑练习第二季】【2-1】
很高兴和大家迎来小分队的第二期~大家在第一期表现都好好呀~要再接再厉。
群里也开始热闹起来了呢~ 虽然花儿落了好多期,考完试一定要补起来~
【精练】
2. Statistician: A financial magazine claimed that its
survey of its subscribers showed that North
Americans are more concerned about their
personal finances than about politics. One
question was: “Which do you think about
more: politics or the joy of earning money?”
This question is clearly biased. Also, the readers
of the magazine are a self-selecting sample.
Thus, there is reason to be skeptical about the
conclusion drawn in the magazine’s survey.
Each of the following, if true, would strengthen the
statistician’s argument EXCEPT:
(A) The credibility of the magazine has been called
into question on a number of occasions.
(B) The conclusions drawn in most magazine
surveys have eventually been disproved.
(C) Other surveys suggest that North Americans are
just as concerned about politics as they are
about finances.
(D) There is reason to be skeptical about the results
of surveys that are biased and
unrepresentative.
(E) Other surveys suggest that North Americans are
concerned not only with politics and
finances, but also with social issues.
【逻辑链】
9. (24497-!-item-!-188;#058&001429)
Beta Corporation claims that it no longer has enough work for the 60 computer data-entry workers that it is laying off. These workers have heard, however, that the company is buying 100 new computers. So the workers concluded that the company's real reason for laying them off is to fill their jobs with lower-paid workers.
Which of the following, if true, would most undermine the workers' conclusion?
(A) Most of the workers being laid off know how to enter data on a number of different computer systems.
(B) Orders for almost all of Beta Corporation products have increased over the past year.
(C) A recent memorandum from the president of Beta Corporation requested that all data generated by the company be stored in computerized form.
(D) Beta Corporation's computer data-entry workers are more experienced and hence more highly paid than people doing comparable work in other companies.
(E) Beta Corporation's new computers will allow its current management and sales staff to perform many of the tasks that the data-entry workers were previously doing.
10. (24867-!-item-!-188;#058&001810)
Which of the following most logically completes the passage?
For the past several years, a certain technology has been widely used to transmit data among networked computers. Recently two data transmission companies, Aptron and Gammatech, have each developed separate systems that allow network data transmission at rates ten times faster than the current technology allows. Although the systems are similarly priced and are equally easy to use, Aptron's product is likely to dominate the market, because __________.
(A) Gammatech has been in the business of designing data transmission systems for several years more than Aptron has
(B) the number of small businesses that need computer networking systems is likely to double over the next few years
(C) it is much more likely that Gammatech's system will be expandable to meet future needs
(D) unlike many data transmission companies, Aptron and Gammatech develop computers in addition to data transmission systems
(E) it is easier for users of the current data transmission technology to switch to Aptron's product than to Gammatech's
11. (24915-!-item-!-188;#058&001837)
Radio stations with radio data system (RDS) technology broadcast special program information that only radios with an RDS feature can receive. Between 1994 and 1996, the number of RDS radio stations in Verdland increased from 250 to 600. However, since the number of RDS-equipped radios in Verdland was about the same in 1996 as in 1994, the number of Verdlanders receiving the special program information probably did not increase significantly.
Which of the following is an assumption on which the argument depends?
(A) Few if any of the RDS radio stations that began broadcasting in Verdland after 1994 broadcast to people with RDS-equipped radios living in areas not previously reached by RDS stations.
(B) In 1996 most Verdlanders who lived within the listening area of an RDS station already had a radio equipped to receive RDS.
(C) Equipping a radio station with RDS technology does not decrease the station's listening area.
(D) In 1996 Verdlanders who did not own radios equipped to receive RDS could not receive any programming from the RDS radio stations that began broadcasting in Verdland after 1994.
(E) The RDS radio stations in Verdland in 1996 did not all offer the same type of programming.
12. (24967-!-item-!-188;#058&001893)
Situation: For five years the Souper restaurant chain has maintained rapid sales growth in Danport, primarily by opening new restaurants that draw patrons away from other restaurants in their vicinity.
Goal: Souper wishes to achieve continued rapid sales growth in Danport over the next two years.
Proposal for consideration: Continue to open new restaurants in Danport during the next two years at the same rate as in the last two years.
In light of the situation, which of the following, if true, most strongly argues that adopting the proposal would be an ineffective way of achieving the goal?
(A) At times at which customers find Souper restaurants too crowded, they often go to other restaurants nearby.
(B) The Souper chain has generally opened new restaurants in locations that are in the vicinity of a large number of other restaurants.
(C) Souper restaurants generally offer a much smaller variety of foods than many of the other restaurants in their vicinity.
(D) Virtually all potential sites for new Souper restaurants in Danport are located in the vicinity of existing Souper restaurants.
(E) Souper restaurants have always offered meals that are somewhat less expensive than meals at restaurants that compete with Souper for patrons.
作者: corneliaflower 时间: 2011-12-9 22:57
KEY:E E A D
bible精解
This problem is more difficult than the previous problem, in part because this is
an Except question. As you recall, in a Strengthen Except question the four
incorrect answers strengthen the argument and the correct answer either has no
effect on the argument or weakens the argument.
The statistician’s statement begins with a variation of the classic LSAT
construction “Some people claim...” As discussed in Chapter Two, when this
construction is used, the author almost always argues against the claim made by
the people. Here, a financial magazine has claimed that a survey proves that
North Americans are more concerned about personal finances than politics. The
statistician attacks two elements of the survey—there was a biased question and
the sampling was faulty—and concludes the magazine’s claim is questionable.
Let us take a closer look at the statistician’s two premises:
1. One question was biased.
The key to understanding this claim is the phrasing of the question in the
magazine: “the joy of earning money.” By describing politics neutrally
but describing earning money as a fun activity, the question
inappropriately suggests to the magazine reader that one activity is more
interesting than the other. This bias undermines the integrity of the
survey.
2. The sample was self-selecting.
A self-selecting sample is one in which individuals decide whether to
participate. As you might expect, only those interested in the topic tend
to participate and this creates a bias in the results. Because the survey
was of subscribers to a financial magazine and not of the general North
American population, those participating in sample are not necessarily
representative of North Americans and thus the magazine cannot
reliably draw a conclusion about North Americans.
Hence, the statistician’s position appears reasonably strong. Nonetheless, you
are asked to eliminate four answers that will strengthen it further.
Earlier in this chapter we mentioned that the test makers believe in the validity
of surveys, polls, etc. This question does not affect that position; in this situation
the survey itself is the topic of discussion. Normally, that is not the case, and
unless a survey or poll is shown to be questionable, you can typically accept the
results knowing that the test makers believe survey results are valid.
Answer choice (A): This answer asserts that the magazine has credibility issues
and thereby supports the conclusion that there should be skepticism regarding
the magazine’s activities.
Answer choice (B): This answer attacks the integrity of magazine surveys, and
therefore supports the idea that there is reason to be skeptical of this magazine
survey. Frankly, this is a weak answer because the validity of surveys in other
magazines do not necessarily reflect on the validity of this magazine’s survey.
Nonetheless, only about five percent of test takers select this answer, as most
people are able to recognize the intent of the test makers.
Answer choice (C): This answer supports the argument because other surveys
suggest that North Americans are not more concerned about finances than
politics. Because this counters the claim of the magazine, the answer supports
the statistician’s conclusion that there is reason to be skeptical of the magazine’s
survey.
Answer choice (D): Because the statistician has shown the survey to be biased
and unrepresentative, this answer choice supports the statistician’s conclusion.
Answer choice (E): This is the correct answer. The answer has no impact on the
statistician’s argument because a third topic—social issues—was not part of the
magazines’ survey, nor does this answer suggest anything about the preference
of North Americas for finance or politics. Because the answer has no impact, it
is correct in a StrengthenX question.
作者: fox0923 时间: 2011-12-9 23:10
花儿好快啊,我来占~~
精练---------------------39s------------------------support
P: The financial magazine reports that North Americans concern their personal finance more than politics.
B: This financial magazine is a self-selecting sampled magazine.
B: And the question of this report is "Do you think more about personal finance or politics?
C: Therefore, all the premised stated above concludes that this magazine's report is not convincible.
Prephrase: This magazine is very biased to report any political issues other than financial news.
找一个irrelevant/weaken的就行了~~
Analysis:
(A) The credibility of the magazine has been called
into question on a number of occasions.-------------------------------------------a number of occasions have been encountered troubles can be the useful samples to illustrate this result is also wrong.
(B) The conclusions drawn in most magazine
surveys have eventually been disproved.-------------------------------------------most magazine surveys include this financial magazine, then it proves that this result of survey might be also disproved.
(C) Other surveys suggest that North Americans are
just as concerned about politics as they are
about finances.--------------------------------------------------------------------------OTher surveys' result is opposed to this result of survey, then support.
(D) There is reason to be skeptical about the results
of surveys that are biased and
unrepresentative.-----------------------------------------------------------------------There's reason to be skeptical that the results should be biased?这个感觉好像是support。
(E) Other surveys suggest that North Americans are
concerned not only with politics and
finances, but also with social issues.------------------------------------------------social issues here is irrelevant. The choice can be "North Americans are concerned not only with fiances, but also with politics."这样的话可以看作是support。
小总:
1. One question was biased.
The key to understanding this claim is the phrasing of the question in the magazine: “the joy of earning money.”---这个部分根本忽略没看了~~
By describing politics neutrally but describing earning money as a fun activity, the question inappropriately suggests to the magazine reader that one activity is more interesting than the other. This bias undermines the integrity of the survey.
2. The sample was self-selecting.
A self-selecting sample is one in which individuals decide whether to participate. As you might expect, only those interested in the topic tend to participate and this creates a bias in the results. Because the survey was of subscribers to a financial magazine and not of the general North American population, those participating in sample are not necessarily representative of North Americans and thus the magazine cannot reliably draw a conclusion about North Americans.
support/weaken的时候就考虑这两个方面就好了~~
作者: corneliaflower 时间: 2011-12-9 23:16
花儿好快啊,我来占~~
-- by 会员 fox0923 (2011/12/9 23:10:49)
嘻嘻 水一下~
因为明天要考试 要早点发完赶紧看书去
作者: zz42050524 时间: 2011-12-10 00:04
A magazine survey infers that Americans are more concern about finance. The evidence is that when asked which do you more concern about personal earning or policy, Americans answered earning.
The conclusion is that the result of the survey is suspect.
Pre: 文中论据 personal earning和finance不是一个概念,所以weaken这个点就是加强
(A) The credibility of the magazine has been called
into question on a number of occasions. 调查样本有问题,这是一个support
(B) The conclusions drawn in most magazine 直接说其他调查结果不同,support
surveys have eventually been disproved.
(C) Other surveys suggest that North Americans are 这里也是在说调查错误,support
just as concerned about politics as they are
about finances.
(D) There is reason to be skeptical about the results 说调查没有代表性,support
of surveys that are biased and
unrepresentative.
(E) Other surveys suggest that North Americans are 这个是无关,social issues和调查的结论无关,是答案
concerned not only with politics and
finances, but also with social issues.
作者: 南瓜0729 时间: 2011-12-10 00:36
占位 有一个星期没做练习了 新的一期 要继续啦~~!
作者: balapupu 时间: 2011-12-10 11:49
P: A magazine show that NA people are more concerned about their own financial problems than the politics?question: F&, do you prefer? since the magazine subscribers are self selecting sample?the conclusion of the magazine is biased.
Support Expect:
(A) The credibility of the magazine has been called
into question on a number of occasions.-->support
(B) The conclusions drawn in most magazine
surveys have eventually been disproved. -->support
(C) Other surveys suggest that North Americans are
just as concerned about politics as they are
about finances. -->support
(D) There is reason to be skeptical about the results
of surveys that are biased and
unrepresentative. -->support
(E) Other surveys suggest that North Americans are
concerned not only with politics and?R
finances, but also with social issues.
作者: xeyyxzty 时间: 2011-12-10 13:41
1.Statistician: Thus, Each of the following, if true, would strengthen the statistician’s argument EXCEPT:background inforamtion:A financial magazine claimed that its survey of its subscribers showed that North Americans are more concerned about their personal finances than about politics.
premise:One question was: “Which do you think about more: politics or the joy of earning money?” This question is clearly biased. Also, the readers of the magazine are a self-selecting sample.
conclusion:there is reason to be skeptical about the conclusion drawn in the magazine’s survey.
(A) The credibility of the magazine has been called into question on a number of occasions.
--irrelevant
(B) The conclusions drawn in most magazine surveys have eventually been disproved.
--support
(C) Other surveys suggest that North Americans are just as concerned about politics as they are about finances.
--support
(D) There is reason to be skeptical about the results of surveys that are biased and unrepresentative.
--support
(E) Other surveys suggest that North Americans are concerned not only with politics and finances, but also with social issues.
----irrelevant
in a Strengthen Except question the four incorrect answers strengthen the argument and the correct answer either has no effect on the argument or weakens the argument.
2.
premise:.Beta Corporation claims that it no longer has enough work for the 60 computer data-entry workers that it is laying off. These workers have heard, however, that the company is buying 100 new computers.
conclusion: the workers concluded that the company's real reason for laying them off is to fill their jobs with lower-paid workers.
there is no link between the number of computers and that of workers
(A) Most of the workers being laid off know how to enter data on a number of different computer systems.
--irrelevant
(B) Orders for almost all of Beta Corporation products have increased over the past year.
--irrelevant
(C) A recent memorandum from the president of Beta Corporation requested that all data generated by the company be stored in computerized form.
--irrelevant
(D) Beta Corporation's computer data-entry workers are more experienced and hence more highly paid than people doing comparable work in other companies.
--support
(E) Beta Corporation's new computers will allow its current management and sales staff to perform many of the tasks that the data-entry workers were previously doing.
--right
3.background information:For the past several years, a certain technology has been widely used to transmit data among networked computers. Recently two data transmission companies, Aptron and Gammatech, have each developed separate systems that allow network data transmission at rates ten times faster than the current technology allows.
premise:he systems are similarly priced and are equally easy to use+???
conclusion:Aptron's product is likely to dominate the market
because it doesn't need to change facilities to fit for the new technology
(A) Gammatech has been in the business of designing data transmission systems for several years more than Aptron has
--irrelevant
(B) the number of small businesses that need computer networking systems is likely to double over the next few years
--irrelevant
(C) it is much more likely that Gammatech's system will be expandable to meet future needs
--weaken
(D) unlike many data transmission companies, Aptron and Gammatech develop computers in addition to data transmission systems
--irrelevant
(E) it is easier for users of the current data transmission technology to switch to Aptron's product than to Gammatech's
--right
4.background information:Radio stations with radio data system (RDS) technology broadcast special program information that only radios with an RDS feature can receive.
premise:Between 1994 and 1996, the number of RDS radio stations in Verdland increased from 250 to 600. However, since the number of RDS-equipped radios in Verdland was about the same in 1996 as in 1994,
conclusion:the number of Verdlanders receiving the special program information probably did not increase significantly.
the listeners of each RDS -radio didn't increase
(A) Few if any of the RDS radio stations that began broadcasting in Verdland after 1994 broadcast to people with RDS-equipped radios living in areas not previously reached by RDS stations.
--irrelevant--right
(B) In 1996 most Verdlanders who lived within the listening area of an RDS station already had a radio equipped to receive RDS.
--irrelevant
(C) Equipping a radio station with RDS technology does not decrease the station's listening area.
--irrelevant
(D) In 1996 Verdlanders who did not own radios equipped to receive RDS could not receive any programming from the RDS radio stations that began broadcasting in Verdland after 1994.
--right
(E) The RDS radio stations in Verdland in 1996 did not all offer the same type of programming.
--irrelevant
5.Situation: For five years the Souper restaurant chain has maintained rapid sales growth in Danport, primarily by opening new restaurants that draw patrons away from other restaurants in their vicinity.
Goal: Souper wishes to achieve continued rapid sales growth in Danport over the next two years.
Proposal for consideration: Continue to open new restaurants in Danport during the next two years at the same rate as in the last two years.
(A) At times at which customers find Souper restaurants too crowded, they often go to other restaurants nearby.--slightly weaken,这个要在D之后才成立
(B) The Souper chain has generally opened new restaurants in locations that are in the vicinity of a large number of other restaurants.
--irrelevant
(C) Souper restaurants generally offer a much smaller variety of foods than many of the other restaurants in their vicinity.
--irrelevant
(D) Virtually all potential sites for new Souper restaurants in Danport are located in the vicinity of existing Souper restaurants.
-- weaken
(E) Souper restaurants have always offered meals that are somewhat less expensive than meals at restaurants that compete with Souper for patrons.
--irrelevant
作者: wangyiwen1005 时间: 2011-12-10 21:12
精练
1.58s
2.BG: A finance magzine show a survey that NA people are more concern about the personal finance than politics.
P: The survey questions are biased. Subjects are seleted by it.
C: The result is skeptical.
P: 还想不到...support反面是weaken吗?
Analysis:
(A) The credibility of the magazine has been called into question on a number of occasions.-------------------------------------support相反
(B) The conclusions drawn in most magazine surveys have eventually been disproved.-------------------------------------------support相反
(C) Other surveys suggest that North Americans are just as concerned about politics as they are
about finances.-------------------------------------------------------------------------support
(D) There is reason to be skeptical about the results of surveys that are biased and
unrepresentative.-----------------------------------------------------------------------support
(E) Other surveys suggest that North Americans are concerned not only with politics and
finances, but also with social issues.------------------------------------------------未提及的新事物si无关,正确
作者: 风无衣 时间: 2011-12-11 12:46
【精炼2-1】
47s
background information:a financial magazine carried out a survey which show northern Americans are more concerned about their financial staff than politics
premise:the question in the survey is biased and the sample is not randomly select.
conclusion:the conclusion conducted by the survey is questionable.
(A) The credibility of the magazine has been called
into question on a number of occasions.——support
(B) The conclusions drawn in most magazine
surveys have eventually been disproved.——support
(C) Other surveys suggest that North Americans are
just as concerned about politics as they are
about finances.——support
(D) There is reason to be skeptical about the results
of surveys/ that are biased and unrepresentative.——support
(E) Other surveys suggest that North Americans are
concerned not only with politics and
finances, but also with social issues.——correct
Answer choice (E): This is the correct answer. The answer has no impact on the
statistician’s argument because a third topic—social issues—was not part of the
magazines’ survey, nor does this answer suggest anything about the preference
of North Americas for finance or politics. Because the answer has no impact, it
is correct in a StrengthenX question.
做题的时候思维还是局限在想要找一个直接weaken conclusion的选项~
再次提醒自己:StrengthenX question的做题思路:irrelevant or weaken!!!
作者: winghyy 时间: 2011-12-11 13:19
精炼题 37s——strengthen EXCEPT
A survey of a magazine showed that the subscribers are more concerned about personal finance than politics. It is not correct for two reasons: 1. the suvey question is "Which do you think about more? Politics or earning money".2 The survey sample is self-selecting.
选
A) The credibility of the magazine has been called into question on a number of occasions.
——strengthen
(B) The conclusions drawn in most magazine surveys have eventually been disproved.
——strengthen
(C) Other surveys suggest that North Americans are just as concerned about politics as they are
about finances.
——strengthen
(D) There is reason to be skeptical about the results of surveys that are biased and unrepresentative.
——strengthen
(E) Other surveys suggest that North Americans are concerned not only with politics and
finances, but also with social issues. ——irrelevant
这题的答案看得迷迷糊糊的。。。
作者: winghyy 时间: 2011-12-12 00:01
逻辑链
9. 43s-weaken
The company laid off the data-entry workers because there is no enough work for them. However, the workers know that the company bought 100 new computers. Therefore, the workers claimed that the reason why the company laid them off is to fill their positions with low-paid workers.
Prephase: The 100 new computers are not for data-entry use. OR There is another new automatic data-entry technology
选E
10. 40s-complete
Both A and G have developed separate systems that allow network data transmission at rates 10 times faster than the current technology. A and G prcied the systems at the same price and the sysmtems are both easy to use. A dominated the market,because_____
Prephase: good marketing promotion
选E
11.60s- assumption
The number of RDS radio station increased from 250 to 600. The number of RDS-equipped radios remained same. Therefore, the number of receiving RDS special programs probably did not increase significantly.
Prephase: There is no other channel to receive RDS programs except for radios.
选A
[答案看不明白,要细细研究一下]
12.47s-weaken
Soupers has remained the rapid growth in the last five years by opening new stores. In the next 2 years, Soupers wishes to continue the rapid growth. Therefore, it is suggested that S should continue to open new stores in the next two years as the same rate as in the last 2 years.
Prephase:
There is no other good locations for S to open new stores.
选D
作者: qiuhua01234567 时间: 2011-12-15 14:35
逻辑2-1
background: Statistician: A financial magazine claimed that its survey of its subscribers showed that North Americans are more concerned about their personal finances than about politics
premise: One question was: “Which do you think about more: politics or the joy of earning money?”
This question is clearly biased. Also, the readers of the magazine are a self-selecting sample.
Conclusion: there is reason to be skeptical about the conclusion drawn in the magazine’s survey
(A) The credibility of the magazine has been called
into question on a number of occasions.
(B) The conclusions drawn in most magazine
surveys have eventually been disproved.
(C) Other surveys suggest that North Americans are
just as concerned about politics as they are
about finances.
(D) There is reason to be skeptical about the results
of surveys that are biased and
unrepresentative.
(E) Other surveys suggest that North Americans are
concerned not only with politics and
finances, but also with social issues.
-----------------------------------------------------------correct
Background:Beta Corporation claims that it no longer has enough work for the 60 computer data-entry workers that it is laying off
Premise:These workers have heard, however, that the company is buying 100 new computers.
Conclusion:So the workers concluded that the company's real reason for laying them off is to fill their jobs with lower-paid workers
(A) Most of the workers being laid off know how to enter data on a number of different computer systems.
(B) Orders for almost all of Beta Corporation products have increased over the past year.
(C) A recent memorandum from the president of Beta Corporation requested that all data generated by the company be stored in computerized form.
(D) Beta Corporation's computer data-entry workers are more experienced and hence more highly paid than people doing comparable work in other companies.
(E) Beta Corporation's new computers will allow its current management and sales staff to perform many of the tasks that the data-entry workers were previously doing.
-------------------------------------------------------------------------------------correct
Background:For the past several years, a certain technology has been widely used to transmit data among networked computers
Conclusion: Although the systems are similarly priced and are equally easy to use, Aptron's product is likely to dominate the market
Premise: Recently two data transmission companies, Aptron and Gammatech, have each developed separate systems that allow network data transmission at rates ten times faster than the current technology allows.
(A) Gammatech has been in the business of designing data transmission systems for several years more than Aptron has
(B) the number of small businesses that need computer networking systems is likely to double over the next few years
(C) it is much more likely that Gammatech's system will be expandable to meet future needs
(D) unlike many data transmission companies, Aptron and Gammatech develop computers in addition to data transmission systems
(E) it is easier for users of the current data transmission technology to switch to Aptron's product than to Gammatech's
---------------------------------------------------------------------------correct
Background:Radio stations with radio data system (RDS) technology broadcast special program information that only radios with an RDS feature can receive。 Between 1994 and 1996, the number of RDS radio stations in Verdland increased from 250 to 600.
However, since the number of RDS-equipped radios in Verdland was about the same in 1996 as in 1994, the number of Verdlanders receiving the special program information probably did not increase significantly
(A) Few if any of the RDS radio stations that began broadcasting in Verdland after 1994 broadcast to people with RDS-equipped radios living in areas not previously reached by RDS stations.
---------------------------------------------------------------------------------------correct
(B) In 1996 most Verdlanders who lived within the listening area of an RDS station already had a radio equipped to receive RDS.
(C) Equipping a radio station with RDS technology does not decrease the station's listening area.
(D) In 1996 Verdlanders who did not own radios equipped to receive RDS could not receive any programming from the RDS radio stations that began broadcasting in Verdland after 1994.
(E) The RDS radio stations in Verdland in 1996 did not all offer the same type of programming.
For five years the Souper restaurant chain has maintained rapid sales growth in Danport, primarily by opening new restaurants that draw patrons away from other restaurants in their vicinity.
Souper wishes to achieve continued rapid sales growth in Danport over the next two years
Continue to open new restaurants in Danport during the next two years at the same rate as in the last two years.
A) At times at which customers find Souper restaurants too crowded, they often go to other restaurants nearby.
(B) The Souper chain has generally opened new restaurants in locations that are in the vicinity of a large number of other restaurants.
(C) Souper restaurants generally offer a much smaller variety of foods than many of the other restaurants in their vicinity.
(D) Virtually all potential sites for new Souper restaurants in Danport are located in the vicinity of existing Souper restaurants.
----------------------------------------------------------------correct
(E) Souper restaurants have always offered meals that are somewhat less expensive than meals at restaurants that compete with Souper for patrons.
作者: ugly5552000 时间: 2011-12-16 00:37
1/Background: a financial magazine claims that the north Americans are more concerned about their personal finances than about politics.
Premise: as the survey question is biased, Statistician asserts the conclusion drawn by the magazine is skeptical.
Prephrase: everyone likes the joy of earning money, most people will not connect the question with finance.
E
A: if the credibility of the magazine has a problem, then the conclusion drawn by the magazine might be unreliable.
B: the survey is contained in those surveys which have been called into question.
C: other surveys hold different conclusion
D: the results of survey is skeptical
E: it does not affect the conclusion
2/Background: 60 computer data-entry workers are laid off, because the company claims that there is not enough work for them.
Premise: since the corporation bought 100 new computers, the laid off workers thought the company must want to recruit lower-paid workers.
Prephrase: the new computers were not bought for data-entry job.
E
A: irrelevant
B: irrelevant
C: the order of the president makes the data-entry work increase dramatically, then the 60 workers should not be laid off.
D: irrelevant
E: the work used to be done by data-entry workers is now resigned to other employees, the work left for the data-entry workers is much less.
3/Background: companies A and G developed advanced systems respectively, their systems are similarly priced and equally easy to use.
Premise: A’ s system is likely to dominate the market.
Prephrase: A controls the network dominantly, A’ s system is compatible with the data format.
E
A: not consistent
B: irrelevant
C: not consistent
D: irrelevant
E: although A and G’ s systems are similarly priced and equally easy used, A’ s system is more acceptable than G’ s in regard of users’ habits.
4/Background: the number of radio stations increased dramatically, the number of radios changed little.
Premise: since the number of radios did not change, the program broadcasted did not increase significantly.
Prephrase: since the number of radios did not change, the revenue got from broadcasting cannot support the cost of radio station.
A
A: after 1994, as those newly built radio stations only broadcast to places which the old radio stations broadcast to, instead of those areas not previously reached by the old stations, and the people live in the places must already have the RDS equipped radios, so no new audience, no increase in the raios.
B: it cannot explain why the special programs did not increase a lot if the new stations broadcast to many other areas.
C: irrelevant
D: without RDS equipped radio, no one can receive the special program, this is a fact.
E: irrelevant
5/Background: souper restaurant chain develops very fast, in the next two years, it plans to maintain its rapid growth.
Premise: if it can continue to open new restaurants at the same rate as it did in the last two years, it will keep the rapid growth.
Prephrase: the market in D has saturated.
D
A: not strong enough
B: shows nothing
C: if customers like the food, even a smaller variety of foods are still alright
D: the reason might be if the new opened restaurants are in the vicinity of the existing restaurants, as it has said that the patrons of new restaurants are often drawn away from others restaurant in the vicinity, in this situation, the patrons needed for the new restaurant might not be satisfied.
E: it will make souper’ s restaurants more competitive, then its growth plan can be successful
作者: fkc8568 时间: 2011-12-29 00:59
精炼:50''
P: survey 结论是民众心中$>politics
C:P中结论是错的
问support C?
思路及预测:Except题,必定4个support,1个weaken或无关,找方向和其余4个相反的那个
(A) The credibility called into question (S)
(B) The conclusions disproved (S)most造成一点迷惑
(C) $=politics (S) 因为证明$没有大于politics
(D) skeptical ( S)
(E) 正确,social issues为无关新概念,且没有讲$和politics的关系。同时证明(B)中的most包含了这个survey,是支持
作者: leewonting 时间: 2012-4-18 17:30
107
背景:Statistician: A financial magazine claimed that its survey of its subscribers showed that North Americans are more concerned about their personal finances than politics.
条件:Question is not fairly asked and the readers of the magazine are a self-selecting sample.
结论:There is reason to skeptical about the conclusion drawn in the magazine’s survey
推测:选出weaken statistician的选项;支持的有:这本杂志是财经杂志、问题的形式不够公正;
选E
108
背景:Beta Corporation claims that it no longer has enough work for the 60 computer data-entry workers that it is laying off.
条件:But the workers saw that the company bought 100 new computers
结论:So the workers concluded that they would be replaced by low-paid workers.
推测:电脑能够处理更多的信息,并没有聘请更low-paid的工人
选E
109
背景:For the past several years, a certain technology has widely used to transmit data among networked computers
条件:Aptron and Gammatech have each developed separate systems that allow network data transmission at rates ten times faster than the current technology allows
结论:Aptron is likely to dominate the market.
推测:A has some advantages that G does not have. 更兼容什么的?
选E
110
Radio stations with radio data system (RDS) technology broadcast special program information that only radios with an RDS feature can receive. Between 1994 and 1996, the number of RDS radio stations in Verdland increased from 250 to 600. However, since the number of RDS-equipped radios in Verdland was about the same in 1996 as in 1994, the number of Verdlanders receiving the special program information probably did not increase significantly.
Which of the following is an assumption on which the argument depends?
(A) Few if any of the RDS radio stations that began broadcasting in Verdland after 1994 broadcast to people with RDS-equipped radios living in areas not previously reached by RDS stations.
(B) In 1996 most Verdlanders who lived within the listening area of an RDS station already had a radio equipped to receive RDS.
(C) Equipping a radio station with RDS technology does not decrease the station's listening area.
(D) In 1996 Verdlanders who did not own radios equipped to receive RDS could not receive any programming from the RDS radio stations that began broadcasting in Verdland after 1994.
(E) The RDS radio stations in Verdland in 1996 did not all offer the same type of programming.
背景:Radio stations with radio data system (RDS) technology broadcast special program information that only radios with an RDS can receive.
条件:Between 1994 and 1996, the number of RDS radio stations in Verdland increased from 250 to 600. However the number of RDS-equipped radios in Verdland was about the same in 1996 as in 1994
结论:The number of the Verdlanders receiving the special program information probably did not increase significantly
推测:RDS radio stations 增加那么多,但是有这种功能的收音机却没有增加很多。要支持上面,其他没有RDS技术的收音机,肯定不能接受到这种节目
选D
101
背景:Situation: For 5 years the Souper restaurant chain has maintained rapid sales growth in Danport, primarily by opening new restaurants that draw patrons away from other restaurants in their vincinty
条件:Souper wishes to achieve continued rapid sales growth in Danport over the next two years.
结论:Continued to open new restaurants in Danport during the next two years at the same rates will not succeed
推测:Danport已经饱和了,没有位置了;客人厌倦了
选D
作者: baseboss 时间: 2012-6-3 21:20
1.people are more concerning about personal finacial than politics.
Aegument for the conclusion
presume:
sample is not correct.
question is bias.
(A) The credibility of the magazine has been called
into question on a number of occasions.
-->strengthen
(B) The conclusions drawn in most magazine
surveys have eventually been disproved.
-->strengthen
(C) Other surveys suggest that North Americans are
just as concerned about politics as they are
about finances.
-->strengthen
(D) There is reason to be skeptical about the results
of surveys that are biased and
unrepresentative.
-->strengthen
(E) Other surveys suggest that North Americans are
concerned not only with politics and
finances, but also with social issues.
-->correct.
2
presume:
The purpose of 100 new computer is not date-enter.
(A) Most of the workers being laid off know how to enter data on a number of different computer systems.
-->support
(B) Orders for almost all of Beta Corporation products have increased over the past year.
-->irrelevant
(C) A recent memorandum from the president of Beta Corporation requested that all data generated by the company be stored in computerized form.
-->irrelevant
(D) Beta Corporation's computer data-entry workers are more experienced and hence more highly paid than people doing comparable work in other companies.
-->irrelevant
(E) Beta Corporation's new computers will allow its current management and sales staff to perform many of the tasks that the data-entry workers were previously doing.
-->correct
3presume:Compatibility
(A) Gammatech has been in the business of designing data transmission systems for several years more than Aptron has
-->support
(B) the number of small businesses that need computer networking systems is likely to double over the next few years
-->irrelevant
(C) it is much more likely that Gammatech's system will be expandable to meet future needs
-->support
(D) unlike many data transmission companies, Aptron and Gammatech develop computers in addition to data transmission systems
-->irrelevant
(E) it is easier for users of the current data transmission technology to switch to Aptron's product than to Gammatech's
-->correct
4 RDS-equipped radios is personal monopolize,no sharing.
(A) Few if any of the RDS radio stations that began broadcasting in Verdland after 1994 broadcast to people with RDS-equipped radios living in areas not previously reached by RDS stations.
-->correct
(B) In 1996 most Verdlanders who lived within the listening area of an RDS station already had a radio equipped to receive RDS.
-->irrelevant
(C) Equipping a radio station with RDS technology does not decrease the station's listening area.
-->irrelevant
(D) In 1996 Verdlanders who did not own radios equipped to receive RDS could not receive any programming from the RDS radio stations that began broadcasting in Verdland after 1994.
-->irrelevant
(E) The RDS radio stations in Verdland in 1996 did not all offer the same type of programming
-->irrelevant
5 Market is limitted in Danport.
(A) At times at which customers find Souper restaurants too crowded, they often go to other restaurants nearby.
-->irrelevant
(B) The Souper chain has generally opened new restaurants in locations that are in the vicinity of a large number of other restaurants.
-->irrelevant
(C) Souper restaurants generally offer a much smaller variety of foods than many of the other restaurants in their vicinity.
-->irrelevant
(D) Virtually all potential sites for new Souper restaurants in Danport are located in the vicinity of existing Souper restaurants.
-->maybe correct
(E) Souper restaurants have always offered meals that are somewhat less expensive than meals at restaurants that compete with Souper for patrons.
-->irrelevant
作者: sachsandciti 时间: 2012-6-20 05:57
re
作者: jetyxo 时间: 2012-6-22 19:12
1 35" background: a survey show that NA people pay more attention to money than to politic
conclusion: this survey is unreasonable
reason: the way of asking question is misleading;the people choosed to answer the question are self-selective
support EXCEPT: E irrelevant
--------------------------------------------------------
2 20" background: BC layoff 60 employees because no job for them to do
premise: BC bought 100 computers
conclusion: the people who lose their job said the company just want to employ low-pay person to do their works
weaken? BC bought computers just to ask other workers do more job which were done by those people who lose job
answer:-------->E
----------------------------------------------------
3 21" background: A and G produce same systerms, sell at the same price and all easely to use
fact: A's products will sell more than G's
reason?-------->E
--------------------------------------------------
4 24" background: only radios with RDS can receive information broadcasted by radiostation with RDS
the number of radiostation with RDS increased during 1994-1996
premise: the people who own radios with RDS did not increase
conclusion:the number of people who receiving information of RDS will not increase
assumption? NO people who owned RDS radios can not receive RDS informatino before the increase in RDS radiostations---------> answer: A
---------------------------------------------------------------------------
5 30" premise: S want to increase its sale through opening new restaurent
weaken: the market has been saturated---->D
作者: jiajiajudy 时间: 2012-7-7 02:34
要做了这么几道才开始有感觉,这咋整…………
1.
Each of the following, if true, would strengthen the statistician’s argument EXCEPT:
(A)
(B)
(C)
(D)
(E) Other surveys suggest that North Americans are concerned not only with politics and finances, but also with social issues.
2. Which of the following, if true, would most undermine the workers' conclusion?
(A)
(B)
(C)
(D)
(E) Beta Corporation's new computers will allow its current management and sales staff to perform
3. many of the tasks that the data-entry workers were previously doing. For the past several years, a certain technology has been widely used to transmit data among networked computers. Recently two data transmission companies, Aptron and Gammatech, have each developed separate systems that allow network data transmission at rates ten times faster than the current technology allows. Although the systems are similarly priced and are equally easy to use, Aptron's product is likely to dominate the market, because
(A)
(B)
(C)
(D)
(E) it is easier for users of the current data transmission technology to switch to Aptron's product than to Gammatech's
4. Radio stations with radio data system (RDS) technology broadcast special program information that only radios with an RDS feature can receive. Between 1994 and 1996, the number of RDS radio stations in Verdland increased from 250 to 600. However, since the number of RDS-equipped radios in Verdland was about the same in 1996 as in 1994, the number of Verdlanders receiving the special program information probably did not increase significantly.
Which of the following is an assumption on which the argument depends?
(A)
(B)
(C)
(D) In 1996 Verdlanders who did not own radios equipped to receive RDS could not receive any programming from the RDS radio stations that began broadcasting in Verdland after 1994.
(E)
5. Situation: For five years the Souper restaurant chain has maintained rapid sales growth in Danport, primarily by opening new restaurants that draw patrons away from other restaurants in their vicinity.
Goal: Souper wishes to achieve continued rapid sales growth in Danport over the next two years.
Proposal for consideration: Continue to open new restaurants in Danport during the next two years at the same rate as in the last two years.
In light of the situation, which of the following, if true, most strongly argues that adopting the proposal would be an ineffective way of achieving the goal?
(A)
(B)
(C)
(D) Virtually all potential sites for new Souper restaurants in Danport are located in the vicinity of existing Souper restaurants.
(E)
作者: meabh 时间: 2012-7-9 20:48
从今天起正式加入 一定要坚持下来
精炼
(1) Time: 34s
(2) Category: Weaken
(3) Logical pattern:
Background: A magazine conducted a survey to conclude NA concern more about money
Premise: Question is biased and sample is self-selecting.
Conclusion: Conclusion is skeptical.
(4) Prephrase: Other questions and readers scope.
(5) Analysis:
(A) The credibility of the magazine has been called into question on a number of occasions. ---- Strengthen (magazine self)
(B) The conclusions drawn in most magazine surveys have eventually been disproved. ----Strengthen (industry comparison)
(C) Other surveys suggest that North Americans are just as concerned about politics as they are about finances. ----Strengthen (similar survey)
(D) There is reason to be skeptical about the results of surveys that are biased and unrepresentative. ----Strengthen (credit reason)
(E) Other surveys suggest that North Americans are concerned not only with politics and finances, but also with social issues. ----Irrelevant (the relationship between politics and finance is unclear).
逻辑链
9.(1) Time: 46s
(2) Category: Weaken
(3) Logical pattern:
Background: BC lay off workers
Premise: Company bought 100 computers.
Conclusion: The lay-off is due to workers’ payment.
(4) Prephrase: Other reason. Perhaps new computers don’t need these workers.
(A) Most of the workers being laid off know how to enter data on a number of different computer systems.
(B) Orders for almost all of Beta Corporation products have increased over the past year.
(C) A recent memorandum from the president of Beta Corporation requested that all data generated by the company be stored in computerized form.
(D) Beta Corporation's computer data-entry workers are more experienced and hence more highly paid than people doing comparable work in other companies.
(E) Beta Corporation's new computers will allow its current management and sales staff to perform many of the tasks that the data-entry workers were previously doing.-right
10.(1) Time: 50s
(2) Category: Complete
(3) Logical pattern:
Background: Transition system has been developed
Premise: Similar price and easy to use in two company
Conclusion: A is dominant
(4) Prephrase: A has something different from B.
(A) Gammatech has been in the business of designing data transmission systems for several years more than Aptron has
(B) the number of small businesses that need computer networking systems is likely to double over the next few years
(C) it is much more likely that Gammatech's system will be expandable to meet future needs
(D) unlike many data transmission companies, Aptron and Gammatech develop computers in addition to data transmission systems
(E) it is easier for users of the current data transmission technology to switch to Aptron's product than to Gammatech's-right
11.(1) Time: 34s
(2) Category: Assumption
(3) Logical pattern:
Background: RDS technology broadcast RDS radio.
Premise: More stations were built but radios were similar
Conclusion: Receivers were same.
(4) Prephrase: New stations brings no change.
(A) Few if any of the RDS radio stations that began broadcasting in Verdland after 1994 broadcast to people with RDS-equipped radios living in areas not previously reached by RDS stations.
(B) In 1996 most Verdlanders who lived within the listening area of an RDS station already had a radio equipped to receive RDS.
(C) Equipping a radio station with RDS technology does not decrease the station's listening area.
(D) In 1996 Verdlanders who did not own radios equipped to receive RDS could not receive any programming from the RDS radio stations that began broadcasting in Verdland after 1994.
(E) The RDS radio stations in Verdland in 1996 did not all offer the same type of programming.
12.(1) Time: 36s
(2) Category: Weaken
(3) Logical pattern:
Background: Past years S has good sales in D
Premise:  erhaps S opened many new restaurants brought the success.
Conclusion: Open new in future can bring good sales, too.
(4) Prephrase: Conditions changed or the reason is not the real cause.
(A) At times at which customers find Souper restaurants too crowded, they often go to other restaurants nearby.
(B) The Souper chain has generally opened new restaurants in locations that are in the vicinity of a large number of other restaurants.
(C) Souper restaurants generally offer a much smaller variety of foods than many of the other restaurants in their vicinity.
(D) Virtually all potential sites for new Souper restaurants in Danport are located in the vicinity of existing Souper restaurants.
(E) Souper restaurants have always offered meals that are somewhat less expensive than meals at restaurants that compete with Souper for patrons.
作者: emmasy 时间: 2012-7-10 10:29
1.
the conclusion is not correct, because the question is biased and the person who answer the question are self-selecting sample, too.
a.support
b.support
c.support
d.weaken
e.irrevelant
2.
facts: the company is going to lay off 60 workers and buy 100 computers.
conclusion:the company's real reason for laying them off is to fill their jobs with lower-paid workers.
a.irrevelant
b.irrevelant
c.irrevelant
d.irrevelant
e.weaken
3.
a.weaken
b.irrevelant
c.weaken
d.irrevelant
e.correct
4.
d.correct
5.
d.correct
作者: Donts 时间: 2012-7-20 10:29
1.45''premise:the question in survey is bias, the readers are selected by the magazine itself, the conclusion made by the magazine is bias.
conclusion:there is reason to be skeptical to the conclusion draw by the magazine.
选择E
(A) The credibility of the magazine has been called
into question on a number of occasions.杂志的可靠度不高,加强。
(B) The conclusions drawn in most magazine
surveys have eventually been disproved.大部分杂志可靠度不高,这个杂志可能被囊括,加强。
(C) Other surveys suggest that North Americans are
just as concerned about politics as they are
about finances.加强。
(D) There is reason to be skeptical about the results
of surveys that are biased and unrepresentative.加强
(E) Other surveys suggest that North Americans are
concerned not only with politics and
finances, but also with social issues.与可靠度无关
选择E
2.计时40''
background:the company lay off 60 computer data-entry workers because works are no longer enough.
premise:the company buy 100 new computers.
conclusion:the workers think the company is to use low paying workers to replace them.
推测:用来做别的工作,并非data entry。
选择E
3.计时30''
both A and G are better than before and they made the same progresses in speed, price and ease to use. but A is better than G
推测:A和G在其他方面的比较,A有更大优势,或者G有劣势。
选择E
4.计时30''
radio station with RDS develop a new program that only radios with program can receive. the radio station increase from 1994 to 1996. however the RDS radios are about the same in 1994 and 1996. thus the # of ppl. receive the special radio didnt increase.
选择D
5.计时42''
background:the S restaurant has rapid sales growth in D because it opens new restaurant in patron far away from other restaurant
planpen restaurant at the same rate
goal:continued rapid sales growth.
选择D
问题:9题C有点纠结,11题A不是很明白,12题的D有点纠结。
订正:4/11.选择A
As you recall, in a Strengthen Except question the four
incorrect answers strengthen the argument and the correct answer either has no
effect on the argument or weakens the argument.
作者: wanggang0411 时间: 2012-8-29 14:36
33
Premise; A biased question survey.
Premise: A self-selected sample.
Conclusion: the conclusion is skeptica.
Inference: Since this si a Except quesion, the correct answer should express the information
that does not support the conclusion.
E: This is an irralavent answer, because whether NA are conerned with social issues has
no impact on the question between private finance and politics.
1:44 I need read slowly, and not repetition.
Premise: BC claims that there is no work for the work that is laying off.
Premise: BC is buying 100 new computers.
conclusion: hiring lower-paid workers is the reaons of laying off.
Inference: weaken.
In the first place, workers committed a fallacy of causal oversimplication.
The arugment of workes rests on a correlation between 100 new computers and hiring
lower-paid workers to establish a causality.
The line of reasoning is that because the company is buying 100 new computers,
hiring lower-paid workers is resposible for laying off.
However, the fact that the company is buying 100 new computers is not a good indicator
of recruiting lower-paid workers. It is possible that the new computers significantly
improved the efficience of data-entry so that the company doesn't need so many data-entry
workers any more.
Thus, the resoning of worker is fallacious unless other possible explainations
have been concerned and ruled out.
E: If the current management and sales stuff can perform the work that data-etry workers
were previous doing, this evidence indicates that company doesn't need so many data-entry
resouces any more and eliminates the explaination of recuiting lower-paid wokers.
24
Premise: two computers have similar price and are equally easy to use.
Conclusion: Ap computer is likely to dominate the market
Inference: Ap computer has some advantage, which is quite attractive for market,
that Ga computer doesn't have.
E
38
Premise: the number of RDS radio station increased from 250 to 600;
Premise: the number of RDS-equiped radio was abou the same in 1996 as in 1994;
Conclusion: the number of people who receive speical program probably did not increase;
Inference: The argument rests on a causal assumption that there is a positive
corrdination between the number of people who receive special program and the number of
RDS-equiped radio, which means that the special program can only be transmitted by
RDS equipment.
D: people without RDS-equipped radio cann't receive any programming from RDS radio stations.
OK, D is just a repetition of the first sentence.A: few if any of 极少有, 即使有 也很少
If many people were not covered by Radio station in 1994, more radio station built in 1996 will cause more people receiving the Radio, thus this answer weakened the argument.
41
Premise: A situation indicates that a restaurant chain who want to rapidly increase their sales.
Premise: G wishes to achive continued sales growth rapidaly.
Conclusion: then you should open new restaruant at the same rates as in the last two years.
Inference: There is no more patrons to be drawn away from other restaurants in his vicinity.
D: So if you open your new resturant in the vicinity of your own existing resturants,
in accordance with the station, you won't draw patrons from other restaurants.
作者: 我心匪席 时间: 2012-9-12 12:23
Time: 34”
Background Information: A magazine claimed that NA are more concerned about their personal finances than about politics. One question was: “Which do you think about more: politics or the joy of earning money?” The question is biased.
Premise: Readers of magazine are self-selecting sample.
Conclusion: To be skeptical about the conclusion drawn in the magazine’s survey.
A-----support
B-----support
C-----support
D-----support
E-----right irrelevant
==============================================
Time: 31”
Background Information: Beta Corporation claims that it no longer has enough work for the 60 computer data-entry workers that is laying off.
Premise: However, company is buying 100 new computers.
Conclusion: Company wants to employ lower-paid workers.
Prephase: Those laying off workers are not so efficient.
A-----irrelevant
B-----irrelevant
C-----irrelevant
D-----irrelevant
E-----right weaken
===============================================
Time: 25”
Background Information: A certain technology has been widely used to transmit data among networked computers. A and G have developed separate systems that allow network data transmission at rates ten times faster than the current technology allows.
Premise: The systems are similarly priced and are equally easy to use.
Conclusion: Aptron's product is likely to dominate the market.
Prephase: A’s system can be applied well to current company.
A-----weaken
B-----irrelevant
C-----irrelevant
D-----irrelevant
E-----right explanation
===============================================
Time: 27”
Background Information: RDS helps radio stations broadcast special program that only radios with an RDS feature can receive. Between 1994 and 1996, the number of RDS radio stations in V increased 250 to 600.
Premise: The number of RDS-equipped radios in V was the same in 1996 as in 1994.
Conclusion: The number of V receives the special program information probably did not increase significantly.
A-----right assumption
B-----irrelevant
C-----irrelevant
D-----irrelevant
E-----irrelevant
==============================================
Time: 38”
Background Information: For five years the Souper restaurant chain has maintained rapid sales growth in Danport, primarily by opening new restaurants that draw patrons away from other restaurants in their vicinity.
Premise: Continue to open new restaurants in Danport during the next two years at the same rate as in the last two years.
Conclusion: Souper wishes to achieve continued rapid sales growth in Danport over the next two years.
Prephase:
A-----irrelevant
B-----irrelevant
C-----irrelevant
D-----right weaken
E-----irrelevant
谢谢楼主整理~
作者: penguinsdz 时间: 2012-10-22 11:19
标题: 10.21+【2-1】
11月2日二战 逻辑keep practicing
1. Statistician: A financial magazine claimed that its survey of its subscribers showed that North Americans are more concerned about their personal finances than about politics. One question was: “Which do you think about more: politics or the joy of earning money?” This question is clearly biased. Also, the readers
of the magazine are a self-selecting sample. Thus, there is reason to be skeptical about the conclusion drawn in the magazine’s survey. Each of the following, if true, would strengthen the statistician’s argument EXCEPT:
(A) The credibility of the magazine has been called into question on a number of occasions.
(B) The conclusions drawn in most magazine surveys have eventually been disproved.
(C) Other surveys suggest that North Americans are just as concerned about politics as they are about finances.
(D) There is reason to be skeptical about the results of surveys that are biased and unrepresentative.
(E) Other surveys suggest that North Americans are concerned not only with politics and finances, but also with social issues.
Background Information:
Statistician: A financial magazine claimed that its survey of its subscribers showed that North Americans are more concerned about their personal finances than about politics.
Premise:
One question was: “Which do you think about more: politics or the joy of earning money?” This question is clearly biased. Also, the readers of the magazine are a self-selecting sample.
Conclusion:
Thus, there is reason to be skeptical about the conclusion drawn in the magazine’s survey.
选项分析:选E
2. Beta Corporation claims that it no longer has enough work for the 60 computer data-entry workers that it is laying off. These workers have heard, however, that the company is buying 100 new computers. So the workers concluded that the company's real reason for laying them off is to fill their jobs with lower-paid workers.
Which of the following, if true, would most undermine the workers' conclusion?
(A) Most of the workers being laid off know how to enter data on a number of different computer systems.
(B) Orders for almost all of Beta Corporation products have increased over the past year.
(C) A recent memorandum from the president of Beta Corporation requested that all data generated by the company be stored in computerized form.
(D) Beta Corporation's computer data-entry workers are more experienced and hence more highly paid than people doing comparable work in other companies.
(E) Beta Corporation's new computers will allow its current management and sales staff to perform many of the tasks that the data-entry workers were previously doing.
Background Information:
Beta Corporation claims that it no longer has enough work for the 60 computer data-entry workers that it is laying off.
Premise:
These workers have heard, however, that the company is buying 100 new computers.
Conclusion:
So the workers concluded that the company's real reason for laying them off is to fill their jobs with lower-paid workers.
选项分析:选E
3. Which of the following most logically completes the passage?
For the past several years, a certain technology has been widely used to transmit data among networked computers. Recently two data transmission companies, Aptron and Gammatech, have each developed separate systems that allow network data transmission at rates ten times faster than the current technology allows. Although the systems are similarly priced and are equally easy to use, Aptron's product is likely to dominate the market, because __________.
(A) Gammatech has been in the business of designing data transmission systems for several years more than Aptron has
(B) the number of small businesses that need computer networking systems is likely to double over the next few years
(C) it is much more likely that Gammatech's system will be expandable to meet future needs
(D) unlike many data transmission companies, Aptron and Gammatech develop computers in addition to data transmission systems
(E) it is easier for users of the current data transmission technology to switch to Aptron's product than to Gammatech's
Background Information:
For the past several years, a certain technology has been widely used to transmit data among networked computers.
Conclusion:
Although the systems are similarly priced and are equally easy to use, Aptron's product is likely to dominate the market.
推测(Prephrase: mentally formulate your answer to the question stem):
It is easier for uses to change facilities to fit for the new technology A than to G.
选项分析:选E
4. Radio stations with radio data system (RDS) technology broadcast special program information that only radios with an RDS feature can receive. Between 1994 and 1996, the number of RDS radio stations in Verdland increased from 250 to 600. However, since the number of RDS-equipped radios in Verdland was about the same in 1996 as in 1994, the number of Verdlanders receiving the special program information probably did not increase significantly.
Which of the following is an assumption on which the argument depends?
(A) Few if any of the RDS radio stations that began broadcasting in Verdland after 1994 broadcast to people with RDS-equipped radios living in areas not previously reached by RDS stations.
(B) In 1996 most Verdlanders who lived within the listening area of an RDS station already had a radio equipped to receive RDS.
(C) Equipping a radio station with RDS technology does not decrease the station's listening area.
(D) In 1996 Verdlanders who did not own radios equipped to receive RDS could not receive any programming from the RDS radio stations that began broadcasting in Verdland after 1994.
(E) The RDS radio stations in Verdland in 1996 did not all offer the same type of programming.
Background Information:
Radio stations with radio data system (RDS) technology broadcast special program information that only radios with an RDS feature can receive.
Premise:
Between 1994 and 1996, the number of RDS radio stations in Verdland increased from 250 to 600.
C-Premise:
Since the number of RDS-equipped radios in Verdland was about the same in 1996 as in 1994.
Conclusion:
The number of Verdlanders receiving the special program information probably did not increase significantly.
推测(Prephrase: mentally formulate your answer to the question stem):
Few people did not receive previously but receive now.
选项分析:选A
5. Situation: For five years the Souper restaurant chain has maintained rapid sales growth in Danport, primarily by opening new restaurants that draw patrons away from other restaurants in their vicinity.
Goal: Souper wishes to achieve continued rapid sales growth in Danport over the next two years.
Proposal for consideration: Continue to open new restaurants in Danport during the next two years at the same rate as in the last two years.
In light of the situation, which of the following, if true, most strongly argues that adopting the proposal would be an ineffective way of achieving the goal?
(A) At times at which customers find Souper restaurants too crowded, they often go to other restaurants nearby.
(B) The Souper chain has generally opened new restaurants in locations that are in the vicinity of a large number of other restaurants.
(C) Souper restaurants generally offer a much smaller variety of foods than many of the other restaurants in their vicinity.
(D) Virtually all potential sites for new Souper restaurants in Danport are located in the vicinity of existing Souper restaurants.
(E) Souper restaurants have always offered meals that are somewhat less expensive than meals at restaurants that compete with Souper for patrons.
Background Information:
Situation: For five years the Souper restaurant chain has maintained rapid sales growth in Danport, primarily by opening new restaurants that draw patrons away from other restaurants in their vicinity.
Premise:
Souper wishes to achieve continued rapid sales growth in Danport over the next two years.
Conclusion:
Adopting the proposal would be an ineffective way of achieving the goal.
推测(Prephrase: mentally formulate your answer to the question stem):
The new restaurants are all in their vicinity, they can not draw patrons away from their old restaurants.
选项分析:选D
作者: srafcatt 时间: 2012-10-22 15:25
精炼 strenghten 28s
Fact:sample---->NAmrns pfinance more
support:maybe all NAns read this magazine.
answer:E
A if the magzine is not convincing,then it makes sense that the survey told us a lie.
B most magzines surveys probably include this magzine.
C according to the information the statistation is likely to be correct
D repeat the information offered in the question stem,strenghten
E correct answer.Because this information has nothing to do with the logic here.
--------------------------------------------------------------------------------
逻辑链
1.weaken 25s
no more work---->co. lay off workers
new computers---->worker think co. want lower-paid staff
maybe new computers is automatically working
answer:E
A this answer indicates that the workers are skilled,but didn't mention why their opinion are wrong
B this answer mentions nothing about the reason why workers are wrong
C this indicates that workers are right
D the same as C
E this is the correct answer for the reason that the work can be done by other staff.
2.fill 30s
Fact:data co.A &co.G similar price and equally easy---->A dominate market
maybe have a pretty good service
answer:E
A this answer indicates that G is better than A
B this is an irrelevant information
C the same as A
D cannot see any difference between A and G
E this is the correct answer for this tell the difference between A &G,and proved that A is easier to switch.
3.assumption 28s
RDS radio receive special prog
94-96,
radio station NO. up & radio NO. same--->receiving special prog NO. up
each radio receive more prog
answer:E
A specific cases are not convincing to prove the phenomenon
B it is not compared to the situation in 1994,so it is not useful
C there's nothing mentioned about the reason of the increasing number of the programs
D it is an irrelevant information
E that's the correct answer.that makes perfect sense.
4.weaken 35s
open new restaus--->sales grow
open new restaus--->continue sales growth
diminishing marginal returns can account for that.
answer
A strenghten the expansion
B just repeat the information in the question stem
C it didn't indicate whether expansion is right or not.
D this is the correct answer for it indicates the expansion space is limited.
E strenghten the expansion for the souper restaurant can meet the need with cheaper price
作者: srafcatt 时间: 2012-10-22 15:41
纠错:逻辑链第三题:本题运用双重否定来判断A 是正确的。
双重否定A之后的逻辑如下:
如果远程发送广播可以被接收的话,每增加一个广播站一定会带来收听人的递增(有点像抢地盘)。
然而A 选项的本来意思是广播只能就近被接收,而且广播站都建设在verdland范围内。所以人数增加有限是很正常的。
我的错误在于对分词翻译的不熟悉。我将结论理解成收到广播的数量没有显著增加了。应该是:收听广播的人数没有随广播站的数量增加而显著增加。
the explanation as follows is from sdcar2010: thanks for her help~~~
Necessary assumption. Use negation.
If you negate A), you have:
Few if any of the RDS radio stations that began broadcasting in Verdland after 1994 DID NOT broadcast to people with RDS-equipped radios living in areas not previously reached by RDS stations.
This is a double negative statement. If that is true, then almost all these stations broadcast to people with RDS-equipped radios living in areas not previously reached by RDS stations. In other words, stations reached out to NEW listeners! If so, the conclusion “the number of Verdlanders receiving the special program information probably did not increase significantly” is wrong. Thus, A) is the necessary assumption.
作者: oytt1111 时间: 2012-10-22 19:06
2
Time: 47s
Premise: survey showed NA are more concerned about their personal finances than about politics
Premise: This question is clearly biased.
Premise: readers are self-selecting sample
Conclusion: there is reason to be skeptical about the conclusion drawn in the survey.
Answer: A
9.
Time: 44s
Premise: BP claims to lay off 60 workers because of not enough work.
Premise: Workers heard, BP is buying 100 new computers.
Conclusion: workers concluded that BP’s real reason is to find new lower paid workers.
Answer: E
10
Time: 33s
Premise: Two data transmission companies developed separate better systems
Premise: the systems are similarly priced and equally easy to use.
Conclusion: A product is likely to dominate the market.
Answer: E
11.
Time: 35s
Premise: Radio station with RDS broadcast special program that only RDS radios can receive.
Premise: Between 1994 and 1996, number of RDS stations increased.
Premise: number of RDS radios was about the same
Conclusion: number of people receiving information din not increase.
Answer: B
12.
Time: 57s
Premise: S maintained rapid sales growth in D by drawing patrons away from other rivals
Premise: S wished to have same rapid growth in next two years
Answer: D
作者: gantian 时间: 2012-10-22 21:17
2. Statistician: A financial magazine claimed that its survey of its subscribers showed that North Americans are more concerned about their personal finances than about politics. One question was: “Which do you think about more: politics or the joy of earning money?” This question is clearly biased. Also, the readers of the magazine are a self-selecting sample. Thus, there is reason to be skeptical about the conclusion drawn in the magazine’s survey.
Each of the following, if true, would strengthen the statistician’s argument EXCEPT:
(A) The credibility of the magazine has been called into question on a number of occasions.
(B) The conclusions drawn in most magazine surveys have eventually been disproved.
(C) Other surveys suggest that North Americans are just as concerned about politics as they are
about finances.
(D) There is reason to be skeptical about the results of surveys that are biased and unrepresentative.
(E) Other surveys suggest that North Americans are concerned not only with politics and
finances, but also with social issues.
Background Information:
A financial magazine claimed that its survey of its subscribers showed that North Americans are more concerned about their personal finances than about politics.
Premise:
One question was: “Which do you think about more: politics or the joy of earning money?” This question is clearly biased.
Also, the readers of the magazine are a self-selecting sample.
Conclusion:
Thus, there is reason to be skeptical about the conclusion drawn in the magazine’s survey.
9. (24497-!-item-!-188;#058&001429)
Beta Corporation claims that it no longer has enough work for the 60 computer data-entry workers that it is laying off. These workers have heard, however, that the company is buying 100 new computers. So the workers concluded that the company's real reason for laying them off is to fill their jobs with lower-paid workers.
Which of the following, if true, would most undermine the workers' conclusion?
Background Information:
Beta Corporation claims that it no longer has enough work for the 60 computer data-entry workers that it is laying off.
Premise:
These workers have heard, however, that the company is buying 100 new computers. Conclusion:
So the workers concluded that the company's real reason for laying them off is to fill their jobs with lower-paid workers.
3) 推测(Prephrase: mentally formulate your answer to the question stem):
给出它因
C(做错,正确选项E)
10. (24867-!-item-!-188;#058&001810)
Which of the following most logically completes the passage?
For the past several years, a certain technology has been widely used to transmit data among networked computers. Recently two data transmission companies, Aptron and Gammatech, have each developed separate systems that allow network data transmission at rates ten times faster than the current technology allows. Although the systems are similarly priced and are equally easy to use, Aptron's product is likely to dominate the market, because __________.
Background Information:
For the past several years, a certain technology has been widely used to transmit data among networked computers. Recently two data transmission companies, Aptron and Gammatech, have each developed separate systems that allow network data transmission at rates ten times faster than the current technology allows.
Premise:
the systems are similarly priced and are equally easy to use
because __________.
Conclusion:
Aptron's product is likely to dominate the market,
3) 推测(Prephrase: mentally formulate your answer to the question stem):
给出它因
E
(A) Gammatech has been in the business of designing data transmission systems for several years more than Aptron has(若加强,还需要更多的前提)
(B) the number of small businesses that need computer networking systems is likely to double over the next few years(彻底无关,方向都不对)
(C) it is much more likely that Gammatech's system will be expandable to meet future needs(加强了G,而非加强A)
(D) unlike many data transmission companies, Aptron and Gammatech develop computers in addition to data transmission systems(没有体现A与G的差别)
(E) it is easier for users of the current data transmission technology to switch to Aptron's product than to Gammatech's
?
11. (24915-!-item-!-188;#058&001837)
Radio stations with radio data system (RDS) technology broadcast special program information that only radios with an RDS feature can receive. Between 1994 and 1996, the number of RDS radio stations in Verdland increased from 250 to 600. However, since the number of RDS-equipped radios in Verdland was about the same in 1996 as in 1994, the number of Verdlanders receiving the special program information probably did not increase significantly.
Which of the following is an assumption on which the argument depends?
Background Information:
Radio stations with radio data system (RDS) technology broadcast special program information that only radios with an RDS feature can receive.
Premise:
Between 1994 and 1996, the number of RDS radio stations in Verdland increased from 250 to 600.
since the number of RDS-equipped radios in Verdland was about the same in 1996 as in 1994,
Conclusion:
the number of Verdlanders receiving the special program information probably did not increase significantly.
3) 推测(Prephrase: mentally formulate your answer to the question stem):
有矛盾,给出它因
Defendant题
A 难
(A) Few if any of the RDS radio stations that began broadcasting in Verdland after 1994 broadcast to people with RDS-equipped radios living in areas not previously reached by RDS stations.
(B) In 1996 most Verdlanders who lived within the listening area of an RDS station already had a radio equipped to receive RDS.
(C) Equipping a radio station with RDS technology does not decrease the station's listening area.
(D) In 1996 Verdlanders who did not own radios equipped to receive RDS could not receive any programming from the RDS radio stations that began broadcasting in Verdland after 1994.
(E) The RDS radio stations in Verdland in 1996 did not all offer the same type of programming.
?
12. (24967-!-item-!-188;#058&001893)
Situation: For five years the Souper restaurant chain has maintained rapid sales growth in Danport, primarily by opening new restaurants that draw patrons away from other restaurants in their vicinity.
Goal: Souper wishes to achieve continued rapid sales growth in Danport over the next two years.
Proposal for consideration: Continue to open new restaurants in Danport during the next two years at the same rate as in the last two years.
In light of the situation, which of the following, if true, most strongly argues that adopting the proposal would be an ineffective way of achieving the goal?
Background Information:
无
Premise:
For five years the Souper restaurant chain has maintained rapid sales growth in Danport, primarily by opening new restaurants that draw patrons away from other restaurants in their vicinity.
Continue to open new restaurants in Danport during the next two years at the same rate as in the last two years.
Conclusion:
Souper wishes to achieve continued rapid sales growth in Danport over the next two years.
3) 推测(Prephrase: mentally formulate your answer to the question stem):
Weaken题,给出它因或者举反例(有因无果、无因有果)
D
(A) At times at which customers find Souper restaurants too crowded, they often go to other restaurants nearby.
(B) The Souper chain has generally opened new restaurants in locations that are in the vicinity of a large number of other restaurants.
(C) Souper restaurants generally offer a much smaller variety of foods than many of the other restaurants in their vicinity.
(D) Virtually all potential sites for new Souper restaurants in Danport are located in the vicinity of existing Souper restaurants.
(E) Souper restaurants have always offered meals that are somewhat less expensive than meals at restaurants that compete with Souper for patrons.
作者: jinyueyue 时间: 2012-10-23 16:46
请假+10.22+【2-1】
2.36s
BG:financial magazine question
PRE:1.the question is biased
2.the readers are a self-selecting sample
CON:skeptical about the survey
可能方向:认为调查结果可以成立 我选择:C 正解:E
错误分析:问的是哪一项不能加强结论 E为无关项,必然不能加强
9.20s
BG:b fired 60 computer data-entry workers for no enough work
PRE:b is buying 100 new computers
CON:real reason is for lower-paid workers
可能方向:不是为了省钱才裁员 我选择:E
10.26s
BG:certain tech used to transmit data
PRE:a and g both develop new system 10 times faster then current ones
price and use are the same
CON:a will win
可能方向:a有其他优势 我选择 E
11.25s
BG:rds broadcast special info
PRE:rds number raised in Vland
CON:rds number did not raise in Vlander.
可能方向:假设他们应该成正比例关系 我选择D 应选A
错误分析:D无关 A取非能削弱
12.37s
BG:s sales increased for 5yrs因为开新饭馆
PRE:s 未来两年还想继续盈利
CON:s还应继续开新店
可能方向:说明s盈利不是因为开店,或有因素导致不能开新店 我选择:D
完毕!
作者: Amashine 时间: 2012-11-13 16:52
2、加强EXCEPT
计时:45’
逻辑链
Background Information:
a magazine survey more concern personal finance>politic
Premise:
1? Concern earn money or polictic, ? biase
2 sample self choose
Conclusion:
the result of survey×
推测(Prephrase: mentally formulate your answer to the question stem):
与P无关
选项分析:
A) The credibility of the magazine has been called
into question on a number of occasions.————————————加强
(B) The conclusions drawn in most magazine
surveys have eventually been disproved.————————————加强
(C) Other surveys suggest that North Americans are
just as concerned about politics as they are
about finances. ————————————————加强
(D) There is reason to be skeptical about the results
of surveys that are biased and
unrepresentative. ————————————————加强
(E) Other surveys suggest that North Americans are
concerned not only with politics and
finances, but also with social issues.————————————————无关
9、44’ 削弱
P1:not enough work -> lay off workers
P2:buy new computers
C:workers: the reason is for cheaper workers
猜:the reason is no work, computers for other use
选:
(A) Most of the workers being laid off know how to enter data on a number of different computer systems.————称述事实,无关
(B) Orders for almost all of Beta Corporation products have increased over the past year.————P1 not enough work, 反
(C) A recent memorandum from the president of Beta Corporation requested that all data generated by the company be stored in computerized form.————反,会需要worker,不应该lay off
(D) Beta Corporation's computer data-entry workers are more experienced and hence more highly paid than people doing comparable work in other companies.————无关比较
(E) Beta Corporation's new computers will allow its current management and sales staff to perform many of the tasks that the data-entry workers——与猜测相符,与computer use有关
10、32’ 填空
P1: tech transmit infor
P2: A&G10times faster>common tech
C: A&G same price, use , but A will dominant,
猜:A与G比较,与price,use无关,与speed有关,或A有其它便利
选:
A) Gammatech has been in the business of designing data transmission systems for several years more than Aptron has————历史长短无关
(B) the number of small businesses that need computer networking systems is likely to double over the next few years————A与G无比较
(C) it is much more likely that Gammatech's system will be expandable to meet future needs——反
(D) unlike many data transmission companies, Aptron and Gammatech develop computers in addition to data transmission systems——A与G无比较
(E) it is easier for users of the current data transmission technology to switch to Aptron's product than to Gammatech's——A比G便利
11、52’ 假设
P1:RDS station can receive RDS radio
P2: 1994-1996 RDS station↑RDS raido =
C: RDS infor ↑×
猜:RDS radio 决定 infor, 而不是station决定infor
Radio 满负荷了
(猜错了)
选:
(A) Few if any of the RDS radio stations that began broadcasting in Verdland after 1994 broadcast to people with RDS-equipped radios living in areas not previously reached by RDS stations.——没有扩大范围,去反后削弱
(B) In 1996 most Verdlanders who lived within the listening area of an RDS station already had a radio equipped to receive RDS.——无关
(C) Equipping a radio station with RDS technology does not decrease the station's listening area.——反
(D) In 1996 Verdlanders who did not own radios equipped to receive RDS could not receive any programming from the RDS radio stations that began broadcasting in Verdland after 1994.——重复P1.无关
(E) The RDS radio stations in Verdland in 1996 did not all offer the same type of programming.——无关
12、42’ 削弱
P1: restaurant open new ones , sales↑
goal: maintain sales ↑ next 2 years
PLAN: open new ones
猜:other condition changed
(A) At times at which customers find Souper restaurants too crowded, they often go to other restaurants nearby.————反,支持Open new ones
(B) The Souper chain has generally opened new restaurants in locations that are in the vicinity of a large number of other restaurants.————无关
(C) Souper restaurants generally offer a much smaller variety of foods than many of the other restaurants in their vicinity.——无关
(D) Virtually all potential sites for new Souper restaurants in Danport are located in the vicinity of existing Souper restaurants.——condition changed
(E) Souper restaurants have always offered meals that are somewhat less expensive than meals at restaurants that compete with Souper for patrons.——无关。Price
作者: Shirleyh707 时间: 2012-12-14 11:23
作者: zxppx 时间: 2013-1-17 11:30
2. Statistician: A financial magazine claimed that its
survey of its subscribers showed that North
Americans are more concerned about their
personal finances than about politics. One
question was: “Which do you think about
more: politics or the joy of earning money?”
This question is clearly biased. Also, the readers
of the magazine are a self-selecting sample.
Thus, there is reason to be skeptical about the
conclusion drawn in the magazine’s survey.
A self-selecting sample is one in which individuals decide whether to participate. As you might expect, only those interested in the topic tend to participate and this creates a bias in the results. Because the survey was of subscribers to a financial magazine and not of the general North American population, those participating in sample are not necessarily representative of North Americans and thus the magazine cannot reliably draw a conclusion about North Americans.
Each of the following, if true, would strengthen the
statistician’s argument EXCEPT:
(A) The credibility of the magazine has been called
into question on a number of occasions.
Since the credibility issues of the magazine, the result conducted by the magazine should be treated highly cautiously, thus strengthening the statistician’s argument.
(B) The conclusions drawn in most magazine
surveys have eventually been disproved.
The same to A.
(C) Other surveys suggest that North Americans are
just as concerned about politics as they are
about finances.
The same to A.
(D) There is reason to be skeptical about the results
of surveys that are biased and
unrepresentative.
The same to A.
(E) Other surveys suggest that North Americans are
concerned not only with politics and
finances, but also with social issues.
Yes, in this case, this answer choice actually has no effect on the argument, thus not strengthening the argument. The comparison referred in the argument is between politics and finances, having no relationship with social issues. BA
9. (24497-!-item-!-188;#058&001429)
Beta Corporation claims that it no longer has enough work for the 60 computer data-entry workers that it is laying off. These workers have heard, however, that the company is buying 100 new computers. So the workers concluded that the company's real reason for laying them off is to fill their jobs with lower-paid workers.
Which of the following, if true, would most undermine the workers' conclusion?
(A) Most of the workers being laid off know how to enter data on a number of different computer systems.
Actually, this answer choice strengthens the argument, since the abilities of the workers being laid off are good, thus meaning that their wages are higher than those of people doing comparable work in other companies.
(B) Orders for almost all of Beta Corporation products have increased over the past year.
This argument does not address the reason of the dismissal.
(C) A recent memorandum from the president of Beta Corporation requested that all data generated by the company be stored in computerized form.
This may help address the reason of purchasing 100 new computers, but it has no effect on the reason of the dismissal.
(D) Beta Corporation's computer data-entry workers are more experienced and hence more highly paid than people doing comparable work in other companies.
The same to A.
(E) Beta Corporation's new computers will allow its current management and sales staff to perform many of the tasks that the data-entry workers were previously doing.
Yes, in this case, this answer choice helps to explanation why the 60 computer data-entry workers are fired, because now management and sales staff can do many of the tasks that the 60 workers were previously doing. And the reason has nothing relationship with lower-paid workers. BA
10. (24867-!-item-!-188;#058&001810)
Which of the following most logically completes the passage?
For the past several years, a certain technology has been widely used to transmit data among networked computers. Recently two data transmission companies, Aptron and Gammatech, have each developed separate systems that allow network data transmission at rates ten times faster than the current technology allows. Although the systems are similarly priced and are equally easy to use, Aptron's product is likely to dominate the market, because __________.
(A) Gammatech has been in the business of designing data transmission systems for several years more than Aptron has
Actually, this answer choice seems to weaken the argument, since more experience would make the products of Gammatech more attractive.
(B) the number of small businesses that need computer networking systems is likely to double over the next few years
This has no effect on the comparison between Aptron’s product and Gammatech’s product.
(C) it is much more likely that Gammatech's system will be expandable to meet future needs
The same to A.
(D) unlike many data transmission companies, Aptron and Gammatech develop computers in addition to data transmission systems
The same to B.
(E) it is easier for users of the current data transmission technology to switch to Aptron's product than to Gammatech's
In this case, the answer choice accurately points out an advantage of Aptro’s product, compared with Gammatech’s product. Bingo!
11. (24915-!-item-!-188;#058&001837)
Radio stations with radio data system (RDS) technology broadcast special program information that only radios with an RDS feature can receive. Between 1994 and 1996, the number of RDS radio stations in Verdland increased from 250 to 600. However, since the number of RDS-equipped radios in Verdland was about the same in 1996 as in 1994, the number of Verdlanders receiving the special program information probably did not increase significantly.
Which of the following is an assumption on which the argument depends?
(A) Few if any of the RDS radio stations that began broadcasting in Verdland after 1994 broadcast to people with RDS-equipped radios living in areas not previously reached by RDS stations.
Even though some RDS radio stations after 1994 broadcast to people with RDS-equipped radios living in areas not previously reached by RDS stations, this answer choice does not strengthen the argument, considering the number of RDS-equipped radios in Verdland was about the same in 1996 as in 1994.
At first I did not think about it carefully, thus making the mistake. When I thought it again, I found it was amazing. In this case, the listeners of RDS would increase between 1994 and 1996, because more people who could not receive the special program information before 1994 would receive it after 1994.
(B) In 1996 most Verdlanders who lived within the listening area of an RDS station already had a radio equipped to receive RDS.
It is opposite to the argument.
(C) Equipping a radio station with RDS technology does not decrease the station's listening area.
It is the opposite answer.
(D) In 1996 Verdlanders who did not own radios equipped to receive RDS could not receive any programming from the RDS radio stations that began broadcasting in Verdland after 1994.
Yes, in this case, it strengthens the argument, since it addresses the hole that Verdlanders who did not own redios equipped to receive RDS could receive some programming from the RDS radio stations that began broadcasting in Verdland after 1994 by the improvement of technology (who say?). It seems to violate the first sentence, but if we use the “negation”, we’ll find that it would weaken the argument. BA
I think I lose the principle in the assumption question. The right answer should comply with the argument. However, this answer is actually opposite to the first sentence in the argument. Although it seems to be attractive, it cannot be true!
(E) The RDS radio stations in Verdland in 1996 did not all offer the same type of programming.
This is irrelevant.
12. (24967-!-item-!-188;#058&001893)
Situation: For five years the Souper restaurant chain has maintained rapid sales growth in Danport, primarily by opening new restaurants that draw patrons away from other restaurants in their vicinity.
Goal: Souper wishes to achieve continued rapid sales growth in Danport over the next two years.
Proposal for consideration: Continue to open new restaurants in Danport during the next two years at the same rate as in the last two years.
In light of the situation, which of the following, if true, most strongly argues that adopting the proposal would be an ineffective way of achieving the goal?
(A) At times at which customers find Souper restaurants too crowded, they often go to other restaurants nearby.
This answer choice, in fact, strengthens the argument.
(B) The Souper chain has generally opened new restaurants in locations that are in the vicinity of a large number of other restaurants.
This has no effect on the argument.
(C) Souper restaurants generally offer a much smaller variety of foods than many of the other restaurants in their vicinity.
Even though this answer choice seems to weaken the argument, the sales of Souper restaurants may increase by adopting the plan, if the food in Souper restaurants is much cheaper and more delicious.
(D) Virtually all potential sites for new Souper restaurants in Danport are located in the vicinity of existing Souper restaurants.
In this case, it cast doubt on the plan, since these new restaurants may compete with existing Souper restaurants, thus making the proposal much less valid. BA
(E) Souper restaurants have always offered meals that are somewhat less expensive than meals at restaurants that compete with Souper for patrons.
This actually strengthens the proposal.
作者: pennyz 时间: 2013-3-4 17:40
2:37s
e strengthen except
premise:survey said that NA care more about money than about politics
it is concluded from a simple question
the sample is flawed
conclusion:the conclusion is not fair
weaken or irrelevant
social issues is out of scope
作者: pennyz 时间: 2013-3-4 18:14
1:32s
weaken e
premise:corporateion no enough post for lay-off workers
purchase 100 more computers
conclusion:laid off because hire low paid new workers
rephrase:corporation use computers for other reasons
and won't hire new workers
1;28s
?
e
background: current data trasmating system widely used
premise :G&a new system
fast
similar price
conclusion:A is more successful
rephrase:because it is easier to use
2:52s
assumption
d
...not known
1:41s
d
background: open new store to lure new customers from other restraunt
premise: open new store at the same rate to sustain the growth
conclusion:may not maintain the growth rate
rephrase: may be too many stores at the same place
作者: Feelalive 时间: 2013-7-15 13:55
非常感谢
作者: Elisha728 时间: 2013-9-4 07:14
6'19''
DEEDD
作者: Elisha728 时间: 2013-9-7 11:10
11 (24915-!-item-!-188;#058&001837)
Radio stations with radio data system (RDS) technology broadcast special program information that only radios with an RDS feature can receive. Between 1994 and 1996, the number of RDS radio stations in Verdland increased from 250 to 600. However, since the number of RDS-equipped radios in Verdland was about the same in 1996 as in 1994, the number of Verdlanders receiving the special program information probably did not increase significantly.
Which of the following is an assumption on which the argument depends?
(A) Few if any of the RDS radio stations that began broadcasting in Verdland after 1994 broadcast to people with RDS-equipped radios living in areas not previously reached by RDS stations.
(B) In 1996 most Verdlanders who lived within the listening area of an RDS station already had a radio equipped to receive RDS.
(C) Equipping a radio station with RDS technology does not decrease the station's listening area.
(D) In 1996 Verdlanders who did not own radios equipped to receive RDS could not receive any programming from the RDS radio stations that began broadcasting in Verdland after 1994.
(E) The RDS radio stations in Verdland in 1996 did not all offer the same type of programmingng.
Premise: the # of RDS radio stations increased; the # of RDS-equipped radios is about the same.
Conclusion: the # of people receiving the special program information probably did not increase significantly.
For assumption question, use NEGATION technique.
After negation, the options become as below:
(A) most of the RDS radio stations that began broadcasting in Verdland after 1994 broadcast to people with RDS-equipped radios living in areas not previously reached by RDS stations.
If so, the # of people receiving the special program information probably will definitely increase significantly. The negation of option A hurt the conclusion. Thus, A is the correct answer.
B) in 1996, not many people who lived within the listening area of an RDS station already had a radio equipped to receive RDS.
The conclusion still stand.
C) equipping a radio station with RDS technology does decrease the station’s listening area.
The conclusion still stand.
D) in 1996, people who did not own radios equipped to receive RDS could receive programming from the RDS radio stations that began broadcasting after 1994.
The conclusion still stand.
E) the RDS radio stations in 1996 did all offer the same type of programming.
The conclusion still stand.
作者: sakurasong 时间: 2013-9-21 14:49
EEEAD
11.
BG: Ratio stations with RDS broadcast special program information that only radios with an RDS features can receive.
P1: 94-96, more RDS stations in V
P2: 94-96, same nb of RDS-equipped radios in V
C: the nb of Ver receiving special program did not increase.
a. more stations, Ver could not receive.
作者: jessie92730 时间: 2013-9-21 17:45
【offer大冒险】 2-1
精练
BG: personal finances > politics
P: the question is biased and the readers are self –selecting samples
C: the conclusion of survey is skeptical
猜测选项:人们易受问题误导
答案:E 文中并没有与social issue有关的部分
逻辑链
1. BG:corporation claims no enough work for 60 workers
P: workers- company is buying 100 new computers
C: true reason- employ low-paid workers
猜测选项:new computer- high efficiency
答案:E
2.BG:A and G’s systems are similarly priced and are equally easy to use
P: ?
C: A’s product is likely to dominate the market
猜测选项:有更多电脑装载了A系统,或A系统有什么特殊优点
答案:E
3. BG: RDS programs must be with RDS-equipped radios
P: programs increased but the number of RDS radios remained
C: the number of people receiving RDS programs did not increase
猜测选项:radio 与 people 数量匹配
答案:A
4:BG: For five years the Souper restaurant chain has maintained rapid sales growth in Danport, primarily by opening new restaurants that draw patrons away from other restaurants in their vicinity.
P: Continue to open new restaurants in Danport during the next two years at the same rate as in the last two years.
C: Souper wishes to achieve continued rapid sales growth in Danport over the next two years.
猜测选项:已没有其他竞争者
答案:D
作者: 艾筱豆 时间: 2013-9-21 21:41
P1:question is biased
P2:readers of the magazine are a self-selecting sample
C:the conclusion is questionable
A the credibility of the magazine has been called into question on a number of occasions
-----这个是加强 如果杂志的信任度都不可靠 可以推断出来 杂志的结论biased
B The conclusion draw in most magazine surveys have eventually been disproved
---因为大多数杂志的结论有问题 那么这个杂志的结论也很可能有问题
C other surveys suggest that North Americans are just as concerned about politics as they are about finance
---支持了第一个方向的加持 推出来杂志是biased 从而支持了结论
D There is a reason to be skeptical about the results of the surveys that are biased and representative
-----支持了第二个方向的假设 支持了结论
E other surveys suggest that North American are concerned not only with politics and finance, but also with social issues
---提到了无关的social issue
9.to weak the argument
There are two ways to consider first:
The new 100 computers are for other use not for the law-pay workers
The staff number does not change because the work will be assigned to the staff
10 A and G are similar in many ways but A’s product is more popular
Should find the difference between the two about product
E is best answer
11 1994-1996
Number of RDS stations increased from 250-600
RDS equipped radios do not change
The assumption should explain the most important the increase of the RDS stations will not affect the number increase of the RDS radio
Or
The there is another reason could explain why the number of the radios does not change
作者: stellagee 时间: 2013-9-21 21:55
【offer大冒险】 2-1
BG: A financial magazine showed that NA concern more about finance than politics.
Premise: The question is simple.
The readers of the financial magazine are self-selecting.
Conclusion: The conclusion of the survey is skeptical.
Q: strengthen except
EEEAD
作者: lyrsilvia 时间: 2013-9-22 19:50
E EEBD
1. 45'
P: question is clearly biased. Also, the readers of the magazine are a self-selecting sample.
C: there is reason to be skeptical about the conclusion drawn in the magazine’s survey.
STRENGTHEN EXPECT: the survey is fair; or other irrelevant factors
答案:E
(A) The credibility of the magazine has been called into question on a number of occasions.---------strengthen the skeptical.
(B) The conclusions drawn in most magazine surveys have eventually been disproved.-----------still strengthen the skeptic.
(C) Other surveys suggest that North Americans are just as concerned about politics as they are about finances.-------------also strengthen the skeptical
(D) There is reason to be skeptical about the results of surveys that are biased and unrepresentative.-----------means the skeptical is reasonable. strengthen.
(E) Other surveys suggest that North Americans are concerned not only with politics and finances, but also with social issues.---------CORRECT. what else does North Americans are concerned is not the core question. irrelevant
2. 20'
P: company is buying 100 new computers
C: company's real reason for laying them off is to fill their jobs with lower-paid workers.
UNDERMINE: more computers≠more lower-paid workers
答案:E
(A) Most of the workers being laid off know how to enter data on a number of different computer systems.-----------how are the workers' abilities does not explain more lower-paid workers
(B) Orders for almost all of Beta Corporation products have increased over the past year.--------------whether orders are increased or not does not explain the workers increase or decrease
(C) A recent memorandum from the president of Beta Corporation requested that all data generated by the company be stored in computerized form.--------------how the data generated does not explain the change of employment
(D) Beta Corporation's computer data-entry workers are more experienced and hence more highly paid than people doing comparable work in other companies.-----------------support the workers' conclusion
(E) Beta Corporation's new computers will allow its current management and sales staff to perform many of the tasks that the data-entry workers were previously doing.---------------CORRECT. means more computers≠more workers
3. 20'
P: the systems are similarly priced and are equally easy to use
C: Aptron's product is likely to dominate the market
EXPLAIN: some advantages of A do not exit in G
答案:E
(A) Gammatech has been in the business of designing data transmission systems for several years more than Aptron has-------------states the advantages of G, does not support the conclusion
(B) the number of small businesses that need computer networking systems is likely to double over the next few years--------------states that the market will broaden. a good situation for both A and G
(C) it is much more likely that Gammatech's system will be expandable to meet future needs----------this statement does not explain the advantageous of A
(D) unlike many data transmission companies, Aptron and Gammatech develop computers in addition to data transmission systems-----------------the comparison between A&G and other companies is irrelevant
(E) it is easier for users of the current data transmission technology to switch to Aptron's product than to Gammatech's-------------CORRECT. clearly state the advantages of A
4. 22'
P: the number of RDS radio stations in Verdland increased from 250 to 600; the number of RDS-equipped radios in Verdland was about the same
C: the number of Verdlanders receiving the special program information probably did not increase significantly.
ASSUMPTION: no more radio=no more increase listener
答案:B
(A) Few if any of the RDS radio stations that began broadcasting in Verdland after 1994 broadcast to people with RDS-equipped radios living in areas not previously reached by RDS stations.-----------means almost all RDS radio stations broadcast to people with RDS-equipped radios in areas not previously reached by RDS stations. 这个选项很难懂啊. 但是抓住一点,这个选项没有提及关于number of equipped radio. CORRECT
(B) In 1996 most Verdlanders who lived within the listening area of an RDS station already had a radio equipped to receive RDS.--------------CORRECT. means although the stations increase, since people already have this kind of radio, they will no need to buy one more. thus the number will not increase.
(C) Equipping a radio station with RDS technology does not decrease the station's listening area.----------------what about the station is irrelevant. focus on whether the number of radio increases
(D) In 1996 Verdlanders who did not own radios equipped to receive RDS could not receive any programming from the RDS radio stations that began broadcasting in Verdland after 1994.-------------. support the equipped radios. 取非:V who did not own radios equipped to receive RDS could receive programming from the RDS. this answer violates the premise. incorrect
(E) The RDS radio stations in Verdland in 1996 did not all offer the same type of programming.--------what program they offer does not affect the radio quantity.
===============================================================错选完全是因为粗心啊!!问的是assumption,就是要证明no more radio=no more listeners. B 直接就推翻了这个逻辑链。A取非之后意思是“some RDS radio stations that began broadcasting in Verdland after 1994 broadcast to people with RDS-equipped radios living in areas not previously reached by RDS stations."【有stations向那些有equipped radios但是住在信号范围外的人播放】意思就是原本就有,不需要再买,但实际上确实听众增加
===============================================================
5. 27'
BG: Souper restaurant chain has maintained rapid sales growth in Danport, primarily by opening new restaurants that draw patrons away from other restaurants in their vicinity.
P: Continue to open new restaurants in Danport during the next two years at the same rate as in the last two years.
C: achieve continued rapid sales growth
WEAKEN: open more store≠rapid sales growth
答案:D
(A) At times at which customers find Souper restaurants too crowded, they often go to other restaurants nearby.------------support the plan since more restaurant means more room for guests
(B) The Souper chain has generally opened new restaurants in locations that are in the vicinity of a large number of other restaurants.-----------where the chain opened does not explain whether the plan is effective or not
(C) Souper restaurants generally offer a much smaller variety of foods than many of the other restaurants in their vicinity.-----------what S offers does not explain whether the plan is effective or not
(D) Virtually all potential sites for new Souper restaurants in Danport are located in the vicinity of existing Souper restaurants.-------------CORRECT. this means opening new restaurants will draw patrons away from its own restaurant.
(E) Souper restaurants have always offered meals that are somewhat less expensive than meals at restaurants that compete with Souper for patrons.------------support the plan
作者: dianadai0029 时间: 2013-10-18 16:53
【精炼】40S strengthen except
Situation
BG: survey showed people more concerned about finances than politics
P: questions type; self-selecting sample
C: the question is biased.
Prephase
deny one of the biase that the argument has, eg. sample biase, question biase, etc.
Analysis
(A) The credibility of the magazine has been called
into question on a number of occasions.----strengthen
(B) The conclusions drawn in most magazine
surveys have eventually been disproved.----strengthen, but not as good as A
(C) Other surveys suggest that North Americans are
just as concerned about politics as they are
about finances.---- strengthen
(D) There is reason to be skeptical about the results
of surveys that are biased and
unrepresentative.----strengthen
(E) Other surveys suggest that North Americans are
concerned not only with politics and
finances, but also with social issues.---->correct,irrelevant
【逻辑链】
9. 30S weaken
BG: lay off 60 workers
P: buy 100 new computer---->C: hire low paided workers to replace the previous ones.
Prephase: computers have other uses
Analysis: A) Most of the workers being laid off know how to enter data on a number of different computer systems.--> irrelevant
(B) Orders for almost all of Beta Corporation products have increased over the past year.-->strengthen
(C) A recent memorandum from the president of Beta Corporation requested that all data generated by the company be stored in computerized form.--> correct, the computer will be used for storing
(D) Beta Corporation's computer data-entry workers are more experienced and hence more highly paid than people doing comparable work in other companies.-->strengthen
(E) Beta Corporation's new computers will allow its current management and sales staff to perform many of the tasks that the data-entry workers were previously doing.--> confused option, many of the tasks can't present all the tasks
10. 34S complete
BG: A and G developed different transmission technology that ten times faster than previous one.
P: similar prices, all easy to use
C: A is more likely to suceed
Prephase: other shortcomes that G has, eg. adjustion cost, after-sale services.
AnalysisA) Gammatech has been in the business of designing data transmission systems for several years more than Aptron has--> weaken
(B) the number of small businesses that need computer networking systems is likely to double over the next few years -->irrelevant
(C) it is much more likely that Gammatech's system will be expandable to meet future needs-->weaken
(D) unlike many data transmission companies, Aptron and Gammatech develop computers in addition to data transmission systems-->irrelevant
(E) it is easier for users of the current data transmission technology to switch to Aptron's product than to Gammatech's-->correct
11. 45S assumption
BG: RDS radio only receive RDS program
P: RDS radio stations increased from 1994 to 1996, while RDS radio didn't increase correspondingly
C: receivers didn't increse
Prephase: deny other reasons could lead the receivers increase, eg. people share the equipment.
Analysis:
A) Few if any of the RDS radio stations that began broadcasting in Verdland after 1994 broadcast to people with RDS-equipped radios living in areas not previously reached by RDS stations.--> confused
(B) In 1996 most Verdlanders who lived within the listening area of an RDS station already had a radio equipped to receive RDS.--> irrelevant, has nothing to do with the premise
(C) Equipping a radio station with RDS technology does not decrease the station's listening area.-->irrelevant
(D) In 1996 Verdlanders who did not own radios equipped to receive RDS could not receive any programming from the RDS radio stations that began broadcasting in Verdland after 1994. -->correct, 说明不会有人没有RDS radio 但是收到了节目
(E) The RDS radio stations in Verdland in 1996 did not all offer the same type of programming. --> irrelevant
12. 44S weaken
BG: D restautant grows fast last serveral years by opening restraunant in new location to drive customers from other restaurant.
C: To keep growing, the company should keep opening new restaunts.
Analysis:
(A) At times at which customers find Souper restaurants too crowded, they often go to other restaurants nearby.
(B) The Souper chain has generally opened new restaurants in locations that are in the vicinity of a large number of other restaurants.
(C) Souper restaurants generally offer a much smaller variety of foods than many of the other restaurants in their vicinity.
(D) Virtually all potential sites for new Souper restaurants in Danport are located in the vicinity of existing Souper restaurants.--> correct
(E) Souper restaurants have always offered meals that are somewhat less expensive than meals at restaurants that compete with Souper for patrons.
作者: ELLENGONG2012 时间: 2013-10-23 16:45
Bg : A magazine did a survey about which north American concern more about finance or politics.
Pm: the question is which one do they think more is biased. Also the question is one-selecting question.
C:the result can be skepticle
Pp: we have to manifest the statistics is sufficient and typical. + By describing politics neutrally but describing earning money as a fun activity, the question inappropriately suggests to the magazine reader that one activity is more interesting than the other. This bias undermines the integrity of the survey.
Each of the following, if true, would strengthen the statistician’s argument EXCEPT: 包括无关和削弱
(A) The credibility of the magazine has been called
into question on a number of occasions.----yes weak the survey` s result.
(B) The conclusions drawn in most magazine surveys have eventually been disproved.---be proved enventually strength. the suspect to result is weaken. 这选项是非证明,所以支持
(C) Other surveys suggest that North Americans are
just as concerned about politics as they are
about finances.----weak the result, so strengthen the author`s view.
(D) There is reason to be skeptical about the results
of surveys that are biased and
unrepresentative.----yes, other reason can strengthen the c.这个是加强啊
(E) Other surveys suggest that North Americans are
concerned not only with politics and
finances, but also with social issues.-----so it proved that the survey is biased. Strengthen the argument.。 无关,只说读者对什么感兴趣,却没说关于是否结论值得怀疑
作者: 览物之情 时间: 2013-10-27 05:44
10月26日
1 35s 属于统计枚举类
Statistician: A financial magazine claimed that its
survey of its subscribers showed that North
Americans are more concerned about their
personal finances than about politics.
Premise: One question was: “Which do you think about
more: politics or the joy of earning money?”
This question is clearly biased. Also, the readers
of the magazine are a self-selecting sample.
Conclusion:Thus, there is reason to be skeptical about the
conclusion drawn in the magazine’s survey.
Each of the following, if true, would strengthen the
statistician’s argument EXCEPT:
The sample is well representative.
(A) The credibility of the magazine has been called
into question on a number of occasions.
(B) The conclusions drawn in most magazine
surveys have eventually been disproved.
(C) Other surveys suggest that North Americans are
just as concerned about politics as they are
about finances.
(D) There is reason to be skeptical about the results of surveys that are biased and
unrepresentative.-----correct没说是什么reason repeat the premise
(E) Other surveys suggest that North Americans are
concerned not only with politics and
finances, but also with social issues.
Answer choice (E): This is the correct answer. The answer has no impact on the
statistician’s argument because a third topic—social issues—was not part of the
magazines’ survey, nor does this answer suggest anything about the preference
of North Americas for finance or politics. Because the answer has no impact, it
is correct in a StrengthenX question.
2 29s
Background:Beta Corporation claims that it no longer has enough work for the 60 computer data-entry workers that it is laying off.
Premise:These workers have heard, however, that the company is buying 100 new computers.
Conclusion: So the workers concluded that the company's real reason for laying them off is to fill their jobs with lower-paid workers.
Which of the following, if true, would most undermine the workers' conclusion?
因果推理。因果没有联系。P the now computer do not need the workers to work with.
(A) Most of the workers being laid off know how to enter data on a number of different computer systems.
(B) Orders for almost all of Beta Corporation products have increased over the past year.
(C) A recent memorandum from the president of Beta Corporation requested that all data generated by the company be stored in computerized form.
(D) Beta Corporation's computer data-entry workers are more experienced and hence more highly paid than people doing comparable work in other companies.
(E) Beta Corporation's new computers will allow its current management and sales staff to perform many of the tasks that the data-entry workers were previously doing.-----correct.
3 26s support family
Which of the following most logically completes the passage?
For the past several years, a certain technology has been widely used to transmit data among networked computers. Recently two data transmission companies, Aptron and Gammatech, have each developed separate systems that allow network data transmission at rates ten times faster than the current technology allows. Premise: Although the systems are similarly priced and are equally easy to use,conclusion Aptron's product is likely to dominate the market, because __________.
类似类比推理A and G 有共同点,但是A更好卖。所以要找A的不同点
A) Gammatech has been in the business of designing data transmission systems for several years more than Aptron has
(B) the number of small businesses that need computer networking systems is likely to double over the next few years
(C) it is much more likely that Gammatech's system will be expandable to meet future needs
(D) unlike many data transmission companies, Aptron and Gammatech develop computers in addition to data transmission systems
(E) it is easier for users of the current data transmission technology to switch to Aptron's product than to Gammatech's----correct
4 27s 相关因果
Premise1:Radio stations with radio data system (RDS) technology broadcast special program information that only radios with an RDS feature can receive.
Between 1994 and 1996, the number of RDS radio stations in Verdland increased from 250 to 600.
Premise:However, since the number of RDS-equipped radios in Verdland was about the same in 1996 as in 1994,
Conclusion:the number of Verdlanders receiving the special program information probably did not increase significantly.
Which of the following is an assumption on which the argument depends?
P one RDS can only be used by one people. what if more people use the same one to get programs.
(A) Few if any of the RDS radio stations that began broadcasting in Verdland after 1994 broadcast to people with RDS-equipped radios living in areas not previously reached by RDS stations.----correct
(B) In 1996 most Verdlanders who lived within the listening area of an RDS station already had a radio equipped to receive RDS.----loser
(C) Equipping a radio station with RDS technology does not decrease the station's listening area.-----loser
(D) In 1996 Verdlanders who did not own radios equipped to receive RDS could not receive any programming from the RDS radio stations that began broadcasting in Verdland after 1994.----might
(E) The RDS radio stations in Verdland in 1996 did not all offer the same type of programming.----loser
5 33s
Situation: For five years the Souper restaurant chain has maintained rapid sales growth in Danport, primarily by opening new restaurants that draw patrons away from other restaurants in their vicinity.
Goal: Souper wishes to achieve continued rapid sales growth in Danport over the next two years.
Proposal for consideration: Continue to open new restaurants in Danport during the next two years at the same rate as in the last two years.
In light of the situation, which of the following, if true, most strongly argues that adopting the proposal would be an ineffective way of achieving the goal?
P 类比推理。说明present condition is different from past condition
A) At times at which customers find Souper restaurants too crowded, they often go to other restaurants nearby.
(B) The Souper chain has generally opened new restaurants in locations that are in the vicinity of a large number of other restaurants.
(C) Souper restaurants generally offer a much smaller variety of foods than many of the other restaurants in their vicinity.
(D) Virtually all potential sites for new Souper restaurants in Danport are located in the vicinity of existing Souper restaurants.----correct
(E) Souper restaurants have always offered meals that are somewhat less expensive than meals at restaurants that compete with Souper for patrons.
作者: w.melhere 时间: 2013-11-3 17:33
1. 1'30 对问题理解有误。‘非加强即削弱的’的思维不对。还可能是无关选项。在这里EXCEPT选择的E选项就是无关项。 其他选项都是加强项。
2. 1'52
3.1'17
4.2'30 这道题还是觉得有点难,做完总结了半天,总算是懂了。逻辑并不复杂,但是A句子本身有点难理解。
A:94年后很少有station向‘有RDS-radio但之前住在收不到RDS信号的地方’的人们广播。
也就是说,电台增多了,但广播范围并没有增大。那么还是一样的设备数量,听众人数也就不会大幅增加了。
D:96年V地没有RDS-radio的居民不能收到94年开始广播RDS的电台。重复了文中的限制---RDS只能被RDS-radio收到。不能作为假设。
few if any of 即使有 也很少
5.1‘43
作者: yuehuasunday 时间: 2013-11-22 14:26
2-1
1.
Background: Statistician: A financial magazine claimed that its
survey of its subscribers showed that North
Americans are more concerned about their
personal finances than about politics.
Premise: One
question was: “Which do you think about
more: politics or the joy of earning money?”
This question is clearly biased.
Conclusion: Also, the readers
of the magazine are a self-selecting sample.
Thus, there is reason to be skeptical about the
conclusion drawn in the magazine’s survey.
(E) Correct
2.
Background: Beta Corporation claims that it no longer has enough work for the 60 computer data-entry workers that it is laying off.
Premise: These workers have heard, however, that the company is buying 100 new computers.
Conclusion: So the workers concluded that the company's real reason for laying them off is to fill their jobs with lower-paid workers.
(E) Correct
3.
Background: For the past several years, a certain technology has been widely used to transmit data among networked computers.
Premise: Recently two data transmission companies, Aptron and Gammatech, have each developed separate systems that allow network data transmission at rates ten times faster than the current technology allows.
Conclusion: Although the systems are similarly priced and are equally easy to use, Aptron's product is likely to dominate the market, because
(E) Correct
4.
Background: Radio stations with radio data system (RDS) technology broadcast special program information that only radios with an RDS feature can receive.
Premise: Between 1994 and 1996, the number of RDS radio stations in Verdland increased from 250 to 600
Conclusion: However, since the number of RDS-equipped radios in Verdland was about the same in 1996 as in 1994, the number of Verdlanders receiving the special program information probably did not increase significantly.
(A) Correct
5.
Background: Situation: For five years the Souper restaurant chain has maintained rapid sales growth in Danport, primarily by opening new restaurants that draw patrons away from other restaurants in their vicinity.
Premise: Goal: Souper wishes to achieve continued rapid sales growth in Danport over the next two years.
Conclusion: Proposal for consideration: Continue to open new restaurants in Danport during the next two years at the same rate as in the last two years.
(D) Correct
作者: 苏错 时间: 2013-11-22 20:54
[精练]
Background: a survey about financial magazine
permise:question is biased and readers are doubtful samples
conclusion:skeptical about the survey
weaken permise or other improvement
A.magazine is doubted support
B.irrelevant
C.irrelevant
D.support
E.correct
[逻辑链]
1.
background:no work for workers
permise: company buy 100 computers - so need to save money
conclusion: to employ lower-paid workers lay workers off
weaken:E
2.similarly priced, equally easy to use, then why is A's product more perferable?
E 相对G而言A的优势
3.numbers increase -- number of V didn't
assumption:A 二者差异
4.background: sales growth
conclusion: continue to open new restaurants -- same results?
weaken: D
作者: irenetopia 时间: 2013-12-4 17:25
2. 2:40min
Statistician: A financial magazine claimed that its
survey of its subscribers showed that North
Americans are more concerned about their
personal finances than about politics. One
question was: “Which do you think about
more: politics or the joy of earning money?”
This question is clearly biased. Also, the readers
of the magazine are a self-selecting sample.
Thus, there is reason to be skeptical about the
conclusion drawn in the magazine’s survey.
Each of the following, if true, would strengthen the
statistician’s argument EXCEPT:
逻辑链:C:the survey in this magazine is biased
R:the question and the sample is biased and non-represent
问weaken :也就是说survey是公正的,说他biased从问题和样本出发,那么找一个可以改变这个的因素
(A) The credibility of the magazine has been called
into question on a number of occasions.——support
(B) The conclusions drawn in most magazine
surveys have eventually been disproved.——support
(C) Other surveys suggest that North Americans are
just as concerned about politics as they are
about finances.——support注意:just as 就是一样,跟原来杂志中的more不一样哦,不要搞混了
(D) There is reason to be skeptical about the results
of surveys that are biased and
unrepresentative.——support
(E) Other surveys suggest that North Americans are
concerned not only with politics and
finances, but also with social issues.——weaken!这算是正面评价咩!
【逻辑链】
9. 1:10min
Beta Corporation claims that it no longer has enough work for the 60 computer data-entry workers that it is laying off. These workers have heard, however, that the company is buying 100 new computers. So the workers concluded that the company's real reason for laying them off is to fill their jobs with lower-paid workers.
Which of the following, if true, would most undermine the workers' conclusion?
逻辑链:C:company lays works inorder to employs less paid worker
P:company has no more work,but it buys many new computers
问削弱:因果型结构,问削弱,就是其实不是为了找更便宜的工人,而且真的是没有更多的工作来做了。
那么为什么买电脑?有别的用途。用电脑代替人?选项会出现新电脑的用途。
(A)Most of the workers being laid off know how to enter data on a number of different computer systems.——irrelevant
(B) Orders for almost all of Beta Corporation products have increased over the past year.——irrelevant
(C) A recent memorandum from the president of Beta Corporation requested that all data generated by the company be stored in computerized form.——irrelevant,如果这样的话还要买电脑干嘛?
(D) Beta Corporation's computer data-entry workers are more experienced and hence more highly paid than people doing comparable work in other companies.——irrelevant
(E) Beta Corporation's new computers will allow its current management and sales staff to perform many of the tasks that the data-entry workers were previously doing.——bingo!解雇一部分人,让留下来的其他职位的人填补这个口子,分担工作。
10. 1:42
Which of the following most logically completes the passage?
For the past several years, a certain technology has been widely used to transmit data among networked computers. Recently two data transmission companies, Aptron and Gammatech, have each developed separate systems that allow network data transmission at rates ten times faster than the current technology allows. Although the systems are similarly priced and are equally easy to use, Aptron's product is likely to dominate the market, because __________.
逻辑链:C: A’s product dominate the market than G’s
P: AandG have the same priced and easy to use
问原因:类比,出现一个A比G的优势的选项
(A) Gammatech has been in the business of designing data transmission systems for several years more than Aptron has——这是weaken,砸场子来的吧
(B) the number of small businesses that need computer networking systems is likely to double over the next few years——irrelevant
(C) it is much more likely that Gammatech's system will be expandable to meet future needs——weaken
(D) unlike many data transmission companies, Aptron and Gammatech develop computers in addition to data transmission systems——共同的,irrelevant
(E) it is easier for users of the current data transmission technology to switch to Aptron's product than to Gammatech's——bingo!switch easily!妈妈再也不用担心我的智商!
11. 1:59
Radio stations with radio data system (RDS) technology broadcast special program information that only radios with an RDS feature can receive. Between 1994 and 1996, the number of RDS radio stations in Verdland increased from 250 to 600. However, since the number of RDS-equipped radios in Verdland was about the same in 1996 as in 1994, the number of Verdlanders receiving the special program information probably did not increase significantly.
Which of the following is an assumption on which the argument depends?
逻辑链:C:RDS station up, radio (with RDS) stable
P: radio (with RDS)can receive the special program,only this kind can receive
问assumption:相关因果性,前提说了R和RDS相关联,后者一个在变化时候,一个不再变化,但是前提看来二者是有相关性的,问假设,其实就是问取非削弱的那个选项切断了相关性。选项必须提到因果
(A) Few if any of the RDS radio stations that began broadcasting in Verdland after 1994 broadcast to people with RDS-equipped radios living in areas not previously reached by RDS stations.——irrelevant
(B) In 1996 most Verdlanders who lived within the listening area of an RDS station already had a radio equipped to receive RDS.——irrelevant,问的是R的数据变化
(C) Equipping a radio station with RDS technology does not decrease the station's listening area.——那么为什么RDS还在up呢?
(D) In 1996 Verdlanders who did not own radios equipped to receive RDS could not receive any programming from the RDS radio stations that began broadcasting in Verdland after 1994.——bingo!取非削弱了原题,切断了相关性
(E) The RDS radio stations in Verdland in 1996 did not all offer the same type of programming.——跟两个变量无关
12. 1:20min
Situation: For five years the Souper restaurant chain has maintained rapid sales growth in Danport, primarily by opening new restaurants that draw patrons away from other restaurants in their vicinity.
Goal: Souper wishes to achieve continued rapid sales growth in Danport over the next two years.
Proposal for consideration: Continue to open new restaurants in Danport during the next two years at the same rate as in the last two years.
In light of the situation, which of the following, if true, most strongly argues that adopting the proposal would be an ineffective way of achieving the goal?
逻辑链:C:opened restaurants in danport at the same rate as last two years
P sales up,due to attract partrons from other restaurants.
问weaken:方案的CQ:找到可能影响的因素。
(A) At times at which customers find Souper restaurants too crowded, they often go to other restaurants nearby.——irrelevant
(B) The Souper chain has generally opened new restaurants in locations that are in the vicinity of a large number of other restaurants.——support
(C) Souper restaurants generally offer a much smaller variety of foods than many of the other restaurants in their vicinity.——irrelevant
(D) Virtually all potential sites for new Souper restaurants in Danport are located in the vicinity of existing Souper restaurants.——bingo!相煎何太急!
(E) Souper restaurants have always offered meals that are somewhat less expensive than meals at restaurants that compete with Souper for patrons.——support
作者: cyndichiang 时间: 2014-4-21 13:19
精炼:
1.36‘’
premise:question conducted by magazine is clearly biased and the readers of the magazine are a self-selecting sample
conclusion: there is reason to be skeptical about the conclusion drawn in the magazine’s survey
Q: support EXCEPT
prephrase: 可能会weaken的选项; the readership of the magazine are various enough to represent the whole citizens. the questions are not biased
(A) The credibility of the magazine has been called
into question on a number of occasions.----support; the credibility is suspected
(B) The conclusions drawn in most magazine
surveys have eventually been disproved.---support, this statement strengthens that since the conclusion is suspected thus has been disapoved
(C) Other surveys suggest that North Americans are
just as concerned about politics as they are
about finances.-----虽然选的是E,但是对这个选项有些不确定。;This answer supports the argument because other surveys suggest that North Americans are not more concerned about finances than politics更能说明,premise中的存在的bias更能说明这个survey是有问题的
(D) There is reason to be skeptical about the results
of surveys that are biased and
unrepresentative.----support;指出当出现bias和unrepresent的时候,是能够被合理怀疑的,证明罗正过程的正确
(E) Other surveys suggest that North Americans are
concerned not only with politics and
finances, but also with social issues.-----correct;this statement is irrelevant
逻辑链
2. 40‘’
premise: 60 computer data-entry workers are laying off because of no enough works.
premise: the company is buying 100 new computers
conclusion: the company's real reason for laying them off is to fill their jobs with lower-paid workers
Q:Weaken
prephrase: the cost of 100 new computers is more expensive than the cost of labor看了选项后发现预想错了, 关键点在与fill with lower-paid workers,因此只要说明不是lower-paid workers
3. 22''
premise: A and G have each developed separate systems that are at the same speed and that the systems are similarly priced and are equally easy to use
conclusion: A's product is likely to dominate the market
Q: support
prepharse: there is a difference between A and G and A is superior than G
作者: cyndichiang 时间: 2014-4-21 17:31
逻辑链
3 49‘’
premise:Between 1994 and 1996, the number of RDS radio stations in Verdland increased from 250 to 600
premise:the number of RDS-equipped radios in Verdland was about the same in 1996 as in 1994
conclusion:the number of Verdlanders receiving the special program information probably did not increase significantly
Q:assumption:
prepharse: the increase of station not effect the number of Vers who have equipped RDS
4.39''
premise: For five years the S restaurant chain has maintained rapid sales growth in D, primarily by opening new restaurants
Proposal:Continue to open new restaurants in D during the next two years at the same rate as in the last two years.
conclusion : p wants to achieve continued rapid sales growth in D over the next two years.
Q:weaken
prephrase: other factors can lead to different consequence since the last two year's condition defers from the next two year's
作者: gmatkenny 时间: 2014-5-23 10:42
corneliaflower 发表于 2011-12-9 22:57
KEY:E E A Dbible精解This problem is more difficult than the previous problem, in part because this ...
很给力!
作者: chemhao90 时间: 2014-5-23 20:05
先蚂一下
作者: jolenewjy 时间: 2014-5-29 07:03
9. B: BC Company lay off workers
P: they bought 100 new companies
C: workers were lay off to be refilled with lower pay
Prediction: the 100 company has their own worker.
Choice: E
10. P: AG products are similar priced and easy to use
C: A product likely to dominate the market
Prediction: some features that A has that will attract customers and G does not have.
Choice: E
11. P: Number of equipped radios is the same
C: number of listener to special program did not increase.
Prediction: Everyone who owns the equipment will use it.
Choice: A
12. P: sales growth driven by open more and draw customer from competitor
C: Open more in the next two years
Prediction: market will change
Choice D
作者: BunnyⅡ 时间: 2014-12-17 01:37
精练 结论:people care finance more than politics 错误 because 1 survey problem is unsatisfied;2 survey sample isn't good.
so 结论不可信
NOT strengthen 推测:survey 有道理、选B 无关 B只是一个z相对weak的strengthenE 提出了第三个subject 无关。
逻辑链 1 公司要开除员工因为公司的电脑不够?? 员工听说公司要买100台新电脑,所以公司实际要雇佣lower paid的员工。
问undermine conclusion 推测 公司买电脑不是为了给这类员工用的 选E
2 BG 要用tech来传送信息 两个公司研发了不同的system price he 速度都差不多。但A 比B 销量好 why?
推测 A system 有独特B 没有的特点 选E
3 BG radio via a special system will only be accepted by a special radio. Premise : the number of the radio program has been increased a lot, but the radios haven't increased. Conclusion: number of audience won't increase.
assumption? 人们听radion的时段没有increase 选E
4 BG resterant open new shops to increase sales Goal: continue increase Methodpen new shop as the same rate in the past.
effecive: 没有其他条件的改变。如别的brand 开业竞争 D
作者: MIA926 时间: 2015-10-3 09:40
20151003 CR小分队
【精练】
2. Statistician: A financial magazine claimed that its
survey of its subscribers showed that North
Americans are more concerned about their
personal finances than about politics. One
question was: “Which do you think about
more: politics or the joy of earning money?”
This question is clearly biased. Also, the readers
of the magazine are a self-selecting sample.
Thus, there is reason to be skeptical about the
conclusion drawn in the magazine’s survey.
计时:24.37s
逻辑链:一个杂志就人们对政治和自身财务状况关心程度展开了调查,但是作者认为这个调查结果不准确,因为sample是杂志自己选的
Each of the following, if true, would strengthen the
statistician’s argument EXCEPT:
(A) The credibility of the magazine has been called
into question on a number of occasions. 加强 以前就不准确过很多次
(B) The conclusions drawn in most magazine
surveys have eventually been disproved. 加强 杂志的结论一般都不准确
(C) Other surveys suggest that North Americans are
just as concerned about politics as they are
about finances. 加强 提出一个和杂志结果相反的调查
(D) There is reason to be skeptical about the results
of surveys that are biased and
unrepresentative. 加强 加强统计学家对调查结果的怀疑是正确的
(E) Other surveys suggest that North Americans are
concerned not only with politics and
finances, but also with social issues. 无关… 正确答案
【逻辑链】
9. (24497-!-item-!-188;#058&001429)
Beta Corporation claims that it no longer has enough work for the 60 computer data-entry workers that it is laying off. These workers have heard, however, that the company is buying 100 new computers. So the workers concluded that the company's real reason for laying them off is to fill their jobs with lower-paid workers.
逻辑:公司说没有活给录信息的人,所以解雇员工;但员工说公司新买了100台电脑,所以解雇他们是为了雇佣低薪员工
推测:要给出一个可以解释为什么买了电脑但是没有活的选项
Which of the following, if true, would most undermine the workers' conclusion?
(A) Most of the workers being laid off know how to enter data on a number of different computer systems. 加强worker观点,错误
(B) Orders for almost all of Beta Corporation products have increased over the past year. 无关
(C) A recent memorandum from the president of Beta Corporation requested that all data generated by the company be stored in computerized form. 加强worker观点,错误
(D) Beta Corporation's computer data-entry workers are more experienced and hence more highly paid than people doing comparable work in other companies. 加强worker观点/无关
(E) Beta Corporation's new computers will allow its current management and sales staff to perform many of the tasks that the data-entry workers were previously doing. 正确,这些worker的工作别人可以干,所以就解雇他们了
10. (24867-!-item-!-188;#058&001810)
Which of the following most logically completes the passage? Assumption
For the past several years, a certain technology has been widely used to transmit data among networked computers. Recently two data transmission companies, Aptron and Gammatech, have each developed separate systems that allow network data transmission at rates ten times faster than the current technology allows. Although the systems are similarly priced and are equally easy to use, Aptron's product is likely to dominate the market, because __________.
逻辑链:BG,AG两公司技术都比现有技术好,而且价格相近,都易使用,但是为什么A公司产品会占领市场?
推测:除了排除的两个原因外,给一个A比G好的理由
(A) Gammatech has been in the business of designing data transmission systems for several years more than Aptron has 削弱
(B) the number of small businesses that need computer networking systems is likely to double over the next few years 无关
(C) it is much more likely that Gammatech's system will be expandable to meet future needs 削弱
(D) unlike many data transmission companies, Aptron and Gammatech develop computers in addition to data transmission systems 无关
(E) it is easier for users of the current data transmission technology to switch to Aptron's product than to Gammatech's 正确
11. (24915-!-item-!-188;#058&001837)
Radio stations with radio data system (RDS) technology broadcast special program information that only radios with an RDS feature can receive. Between 1994 and 1996, the number of RDS radio stations in Verdland increased from 250 to 600. However, since the number of RDS-equipped radios in Verdland was about the same in 1996 as in 1994, the number of Verdlanders receiving the special program information probably did not increase significantly.
Which of the following is an assumption on which the argument depends?
逻辑链:BG,RDS station increased, but since RDS radios remain the same, people will receive special programs did not increase.
推测:可能原来有RDS radio 的人收不到,station原来覆盖不到,补gap
(A) Few if any of the RDS radio stations that began broadcasting in Verdland after 1994 broadcast to people with RDS-equipped radios living in areas not previously reached by RDS stations. 正确
(B) In 1996 most Verdlanders who lived within the listening area of an RDS station already had a radio equipped to receive RDS. 削弱
(C) Equipping a radio station with RDS technology does not decrease the station's listening area. 无关
(D) In 1996 Verdlanders who did not own radios equipped to receive RDS could not receive any programming from the RDS radio stations that began broadcasting in Verdland after 1994. 无关
(E) The RDS radio stations in Verdland in 1996 did not all offer the same type of programming. 无关
12. (24967-!-item-!-188;#058&001893)
Situation: For five years the Souper restaurant chain has maintained rapid sales growth in Danport, primarily by opening new restaurants that draw patrons away from other restaurants in their vicinity.
Goal: Souper wishes to achieve continued rapid sales growth in Danport over the next two years.
Proposal for consideration: Continue to open new restaurants in Danport during the next two years at the same rate as in the last two years.
In light of the situation, which of the following, if true, most strongly argues that adopting the proposal would be an ineffective way of achieving the goal?
逻辑:过去五年S通过不断开心点抢顾客获得收益,接下来两年S希望还可以获得相同的收益,所以应该继续开新店
推测:错误类推,过去五年的情况和现在的情况不同,现在很多原来的点都被挤走了,剩下D自己的店,再开新店会和自己造成竞争
(A) At times at which customers find Souper restaurants too crowded, they often go to other restaurants nearby. 加强,错误
(B) The Souper chain has generally opened new restaurants in locations that are in the vicinity of a large number of other restaurants. 加强
(C) Souper restaurants generally offer a much smaller variety of foods than many of the other restaurants in their vicinity. 无关
(D) Virtually all potential sites for new Souper restaurants in Danport are located in the vicinity of existing Souper restaurants. 正确
(E) Souper restaurants have always offered meals that are somewhat less expensive than meals at restaurants that compete with Souper for patrons. 加强
作者: 蚂蚁头头 时间: 2015-10-20 20:44
9. (24497-!-item-!-188;#058&001429)
Beta Corporation claims that it no longer has enough work for the 60 computer data-entry workers that it is laying off. These workers have heard, however, that the company is buying 100 new computers. So the workers concluded that the company's real reason for laying them off is to fill their jobs with lower-paid workers.
Which of the following, if true, would most undermine the workers' conclusion?
p; B lau off the worker since no enough job.
c: worker think that the reason is the wage is high
10. (24867-!-item-!-188;#058&001810)
Which of the following most logically completes the passage?
For the past several years, a certain technology has been widely used to transmit data among networked computers. Recently two data transmission companies, Aptron and Gammatech, have each developed separate systems that allow network data transmission at rates ten times faster than the current technology allows. Although the systems are similarly priced and are equally easy to use, Aptron's product is likely to dominate the market, because __________.
两家公司各自技术相比现在技术而言,都能使信息传送十倍以上。两家公司加个相同且易上手,但是A公司更可能主导市场
11. (24915-!-item-!-188;#058&001837)
Radio stations with radio data system (RDS) technology broadcast special program information that only radios with an RDS feature can receive. Between 1994 and 1996, the number of RDS radio stations in Verdland increased from 250 to 600. However, since the number of RDS-equipped radios in Verdland was about the same in 1996 as in 1994, the number of Verdlanders receiving the special program information probably did not increase significantly.
Which of the following is an assumption on which the argument depends?
Radio Station 增加,Radio with RDS数量没变,所以能收到信息得V人很可能不会明显增加
12. (24967-!-item-!-188;#058&001893)
Situation: For five years the Souper restaurant chain has maintained rapid sales growth in Danport, primarily by opening new restaurants that draw patrons away from other restaurants in their vicinity.
Goal: Souper wishes to achieve continued rapid sales growth in Danport over the next two years.
Proposal for consideration: Continue to open new restaurants in Danport during the next two years at the same rate as in the last two years.
In light of the situation, which of the following, if true, most strongly argues that adopting the proposal would be an ineffective way of achieving the goal?
Since Souper opened new restaurants in D, its sales rapid growth.
Goal: In order to keep rapid sales it plans to open more new restaurants in D.
作者: spencerchan 时间: 2016-1-31 14:59
44’’
P: Based on the interview, North Americans are more concerned about their personal finances than about politics.
C: Question designed/ self-selecting sample.
A,B,C,D: Strengthen
E: CORRECT
1’03’’
P: 60 computer data-entry workers are laying off
P: Company is buying 100 new computers
C: Those laid off employees argue that the real reason is the company want hire low cost employees to replace them
Weaken: Those 100 new computers do not need workers to entry data.??
A: Irrelevant
B: Irrelevant
C: Irrelevant
D: Strengthen
E: CORRECT
47’’
P: A&G products are almost same
C: A is likely to dominate the market
Assumption: A has a unique advantage
A: Opposite
B: Irrelevant
C: Opposite
D: Irrelevant
E: CORRECT
1’12’’ 这道题看不懂,被绕进去了
P: radio stations with radio data system => special programs
P: RDS radio stations increase, but RDS-equipped radios was the same
C: special programs not increase
Assumption: other equipment cannot have RDS
A: ??
B: Irrelevant?
C: Irrelevant
D: CORRECT?
E: Irrelevant
47”
P: S restaurant opened new restaurants that draw customs aways from others=> sales boost
P: S decided to continue open new restaurants
C: Sales cannot maintain the same increased rate
Assumption: The market has been crowded
A: Irrelevant
B: Irrelevant
C: Irrelevant
D: CORRECT
E: Irrelevant
作者: May97 时间: 2018-3-8 12:25
T: 40S
FM: North Americans are more concerned about their personal finances than about politics.
P: Also, the readers of the magazine are a self-selecting sample.
C: Thus, there is reason to be skeptical about the conclusion drawn in the magazine’s survey.
SUPPORT EXPECT: READERS NOT EQUAL TO NORTH AMERICANS
(A) The credibility of the magazine has been called into question on a number of occasions.
(B) The conclusions drawn in most magazine surveys have eventually been disproved.
(C) Other surveys suggest that North Americans are just as concerned about politics as they are about finances.
(D) There is reason to be skeptical about the results of surveys that are biased and unrepresentative.
(E) Other surveys suggest that North Americans are concerned not only with politics and finances, but also with social issues.--->R
T: 30S
P: Beta Corporation claims that it no longer has enough work for the 60 computer data-entry workers that it is laying off.
P: the company is buying 100 new computers.
C: So the workers concluded that the company's real reason for laying them off is to fill their jobs with lower-paid workers.
WEAKEN: DATA IS DIRECTLY RECORDED IN THE COMPUTER.
(A) Most of the workers being laid off know how to enter data on a number of different computer systems.--->IRREVERENT
(B) Orders for almost all of Beta Corporation products have increased over the past year.--->A BIT SUPPORT
(C) A recent memorandum from the president of Beta Corporation requested that all data generated by the company be stored in computerized form.--->SUPPORT
(D) Beta Corporation's computer data-entry workers are more experienced and hence more highly paid than people doing comparable work in other companies.--->IRREVERENT
(E) Beta Corporation's new computers will allow its current management and sales staff to perform many of the tasks that the data-entry workers were previously doing.--->R
T: 33S
BG: For the past several years, a certain technology has been widely used to transmit data among networked computers.
P: Aptron and Gammatech, have each developed separate systems that allow network data transmission at rates ten times faster than the current technology allows.
P: the systems are similarly priced and are equally easy to use,
C: Aptron's product is likely to dominate the market,
FA: AD OR SOME OTHER REASONS, RATHER THAN COST AND USE
(A) Gammatech has been in the business of designing data transmission systems for several years more than Aptron has--->IRREVERENT
(B) the number of small businesses that need computer networking systems is likely to double over the next few years--->IRREVERENT
(C) it is much more likely that Gammatech's system will be expandable to meet future needs--->IRREVERENT
(D) unlike many data transmission companies, Aptron and Gammatech develop computers in addition to data transmission systems--->IRREVERENT
(E) it is easier for users of the current data transmission technology to switch to Aptron's product than to Gammatech's--->R
T: 50S
P: Radio stations with radio data system (RDS) technology broadcast special program information that only radios with an RDS feature can receive.
P:Between 1994 and 1996, the number of RDS radio stations in Verdland increased from 250 to 600.
P: However, since the number of RDS-equipped radios in Verdland was about the same in 1996 as in 1994,
C: the number of Verdlanders receiving the special program information probably did not increase significantly.
FA:
(A) Few if any of the RDS radio stations that began broadcasting in Verdland after 1994 broadcast to people with RDS-equipped radios living in areas not previously reached by RDS stations.--->R
(B) In 1996 most Verdlanders who lived within the listening area of an RDS station already had a radio equipped to receive RDS.--->IRREVERENT
(C) Equipping a radio station with RDS technology does not decrease the station's listening area.--->irreverent
(D) In 1996 Verdlanders who did not own radios equipped to receive RDS could not receive any programming from the RDS radio stations that began broadcasting in Verdland after 1994.--->THE SAME AS P
(E) The RDS radio stations in Verdland in 1996 did not all offer the same type of programming.--->IIEVERENT
这题A选项有点绕。。看了半天才看明白。。因为C选项也有listening area,刚开始差点选了C
T: 46S
BG: For five years the Souper restaurant chain has maintained rapid sales growth in Danport, primarily by opening new restaurants that draw patrons away from other restaurants in their vicinity.
C: Souper wishes to achieve continued rapid sales growth in Danport over the next two years.
P: Continue to open new restaurants in Danport during the next two years at the same rate as in the last two years.
WEAKEN: THE SCALE OF THE RESTAURANT REACHES PEAK
(A) At times at which customers find Souper restaurants too crowded, they often go to other restaurants nearby.--->SUPPORT
(B) The Souper chain has generally opened new restaurants in locations that are in the vicinity of a large number of other restaurants.--->IRREVERENT
(C) Souper restaurants generally offer a much smaller variety of foods than many of the other restaurants in their vicinity.--->IRREVERENT
(D) Virtually all potential sites for new Souper restaurants in Danport are located in the vicinity of existing Souper restaurants.--->R
(E) Souper restaurants have always offered meals that are somewhat less expensive than meals at restaurants that compete with Souper for patrons.--->IRREVERENT
作者: May97 时间: 2018-3-8 12:30
精炼
Answer choice (B): This answer attacks the integrity of magazine surveys, and
therefore supports the idea that there is reason to be skeptical of this magazine
survey. Frankly, this is a weak answer because the validity of surveys in other
magazines do not necessarily reflect on the validity of this magazine’s survey.
作者: echo-LUO 时间: 2018-10-3 10:42
精练一个测试结果显示,美国人比关注政治,更关注自己的经济——>这个测试不好,第一,问题提的有偏差,第二,样本限制——>所以结果并不值得参考
(A) The credibility of the magazine has been called into question on a number of occasions.
(B) The conclusions drawn in most magazine surveys have eventually been disproved.
(C) Other surveys suggest that North Americans are just as concerned about politics as they are about finances.
(D) There is reason to be skeptical about the results of surveys that are biased and unrepresentative.
(E) Other surveys suggest that North Americans are concerned not only with politics and finances, but also with social issues. 无关。
逻辑链
公司说没有工作了,所以开除了一堆电脑录入工人——>公司又买了大批电脑——>工人:才不是没有工作,只是公司想招募更便宜的工人
(A) Most of the workers being laid off know how to enter data on a number of different computer systems.
(B) Orders for almost all of Beta Corporation products have increased over the past year.
(C) A recent memorandum from the president of Beta Corporation requested that all data generated by the company be stored in computerized form.
(D) Beta Corporation's computer data-entry workers are more experienced and hence more highly paid than people doing comparable work in other companies.
(E) Beta Corporation's new computers will allow its current management and sales staff to perform many of the tasks that the data-entry workers were previously doing. 新电脑使现有的人能做被开除的人的工作,而不是换成更廉价的劳动力,成功weaken
两家数据传送公司,效率都是现有水平的十倍,价格和使用难度都相当,当a公司比g公司更能占据市场
(A) Gammatech has been in the business of designing data transmission systems for several years more than Aptron has
(B) the number of small businesses that need computer networking systems is likely to double over the next few years
(C) it is much more likely that Gammatech's system will be expandable to meet future needs
(D) unlike many data transmission companies, Aptron and Gammatech develop computers in addition to data transmission systems
(E) it is easier for users of the current data transmission technology to switch to Aptron's product than to Gammatech's. 说出了a比g优势的地方
rds广播被有rds的器材接受——>近几年rds广播站数量增加,但是rds器材没有增加——>v地的听众没有增加
(A) Few if any of the RDS radio stations that began broadcasting in Verdland after 1994 broadcast to people with RDS-equipped radios living in areas not previously reached by RDS stations. 有可能之前有rds设备的人,不在station的范围内
(B) In 1996 most Verdlanders who lived within the listening area of an RDS station already had a radio equipped to receive RDS.
(C) Equipping a radio station with RDS technology does not decrease the station's listening area.
(D) In 1996 Verdlanders who did not own radios equipped to receive RDS could not receive any programming from the RDS radio stations that began broadcasting in Verdland after 1994.
(E) The RDS radio stations in Verdland in 1996 did not all offer the same type of programming.
某店主要靠新店开张增加客流量——>为了保持后两年增长——>应该采取继续开新店的策略
A) At times at which customers find Souper restaurants too crowded, they often go to other restaurants nearby.
(B) The Souper chain has generally opened new restaurants in locations that are in the vicinity of a large number of other restaurants.
(C) Souper restaurants generally offer a much smaller variety of foods than many of the other restaurants in their vicinity.
(D) Virtually all potential sites for new Souper restaurants in Danport are located in the vicinity of existing Souper restaurants. 店铺满了
(E) Souper restaurants have always offered meals that are somewhat less expensive than meals at restaurants that compete with Souper for patrons.
作者: 云栈 时间: 2019-11-12 14:14
1. 36s
P: 统计学家:一家金融杂志声称,对订户的调查显示,北美人更关心自己的个人理财,而不是政治。
P: 一个问题是:“您还考虑什么:政治还是赚钱的乐趣?”这个问题显然带有偏见。
P:同样,杂志的读者是一个自我选择的样本。
C: 因此,有理由怀疑该杂志的调查结论。
推测:无关选项;或者是证明问题没有biased或者是杂志读者涉及范围很广的
选项分析:E
A: 说明杂志可信度不高,调查结果不准确
B:大部分杂志的调查都不可信,间接说明这个可能也不可信
C:有其他的调查得到不一样的结果,间接说明这个杂志的结果遭到质疑
D: 补充说明如果一个调查是biased,那么就有理由质疑它
E: 与原题无关
2. 20s
P: Beta Corporation声称,对于即将解雇的60名计算机数据输入工作者来说,它的工作量不再足够。
P: 但是,这些工人听说该公司正在购买100台新计算机。
C: 因此,工人得出结论,公司解雇他们的真正原因是用低薪工人来填补他们的工作。
推测:证明买了100个电脑可能是给其他position的人换的,而不是要招新的岗位
选项分析:E
3. 22s
P: 在过去的几年中,某种技术已被广泛用于在联网计算机之间传输数据。
P:最近,两家数据传输公司Aptron和Gammatech各自开发了独立的系统,这些系统允许网络数据传输的速率比当前技术所允许的快十倍。
C:尽管这些系统的价格相似并且易于使用,但是Aptron的产品很可能会占领市场,因为__________。
推测:任何证明A比S更受众的原因;比如A的广告打得好;A的信誉高
选项分析:E
4. 40s
P: 具有无线电数据系统(RDS)技术的广播电台广播特殊节目信息,只有具有RDS功能的无线电才能接收。
P: 在1994年到1996年之间,韦德兰的RDS广播电台的数量从250个增加到600个。
C: 但是,由于1996年韦德兰配备RDS的无线电的数量与1994年大致相同,因此接受特别节目信息的韦德兰德的数量可能没有明显增加。
推测:没有更多的人聚在一起听同一个radio
选项分析:A
【这题选了D, 主要因为A太绕了没看懂。原题逻辑链没看懂。原题应该是再说station增加了,但是设备数量没增加,所以说明接受的人数没增加。A补充了gap说明,即使station增加了,但是覆盖面没有增加】
5. 1min22s
P: 状况:五年来,Souper连锁餐厅在Danport保持了快速的销售增长,主要是通过开设新餐厅来吸引顾客远离附近的其他餐厅。
P: 目标:Souper希望在未来两年内实现Danport持续快速的销售增长。
P: 供考虑的建议:未来两年继续以与过去两年相同的速度在丹波特开设新餐厅。
推测:市场对餐厅的需求量已经临近饱和
选项分析:D
作者: Lincy123 时间: 2020-6-2 08:26
P:样本全部为单独挑选杂志读者、问题为:你考虑政治更多还是赚钱的快乐
C:杂志的调查结果-北美人更关心钱-不可信
Weaken:读者都是北美人
A) 杂志的信用有问题 strengthen
B) 大部分杂志的结果都是假的。Strengthen
C) 别的调查说北美的人对钱和政治同样关心。无关 correct(x)
D)调查结果很可疑。Strengthen
E) 别的调查表明北美的人关心其他因素。Strengthen(x)correct
社会事务和题目无关,也没有表明北美的人更偏好什么,对argument无影响
P:公司买了100台新电脑
C:被裁的员工觉得公司就是想招更便宜的员工
Weaken:新电脑不需要人工
A) 被裁的员工知道怎么在不同电脑系统输入数据。无关
B) 订单上升。无关
C) 所有电脑产生的数据以编程模式储存,无关
D)BC公司的员工薪水更高,无关
E) 新电脑让现有员工做被裁员工做的事情。Correct
C:价格和使用容易度一样,A的产品占领市场
P:? 可能效率、速度等
A) G更有经验,weaken
B) 同类公司数量会翻倍。无关
C) G的系统满足未来需求。Weaken
D)A和G也做电脑。无关
E) 现有用户更容易切换到A,correct
P:RDS基站增加,有RDS收音机的人数没增加
C:接受特殊节目的人数不会增加
Assumption:接收program的人和有RDS radio的人的关系,基站和人数的关系
A) 很少1994年获基站广播给有RDS收音机,之前没有接触基站的人。Correct
B) 1996年大部分住在电台接收范围的人本来就有RDS收音机
C) 有RDS技术不会降低收听范围
D)在1996年没有RDS收音机的人听不了任何广播
E) 基站没有全部提供相同的节目。无关
P:开新餐厅吸引隔壁餐厅的顾客
C:销售增长
A) 顾客觉得S餐厅太挤了,他们会去其他餐厅。无关
B) S常常把新店开在其他餐厅隔壁,无关
C) S比隔壁餐厅提供的餐点种类更少样。无关
D)所有能开新餐厅的地方都在现有的S餐厅附近。Correct
E) S的餐点更便宜。
欢迎光临 ChaseDream (https://forum.chasedream.com/) |
Powered by Discuz! X3.3 |